SlideShare a Scribd company logo
1 of 90
Home4 Why Europe and not China?
1. Why does Landes think that China would not have developed
an industrial revolution on its own? (Landes 2006 “Why Europe
and the West? Why not China?” is posted on file)
2. Why does he think that China failed to learn new
technologies from Europeans in the period after 1500?
3. In Landes’ view, what did Europe have that China lacked?
That is, what did Europe have that permitted it to have an
industrial revolution?
4. What does Pomeranz say about the factors that Landes
identifies as the crucial features of European society that
permitted it to have an industrial revolution? Why does he say
that these features did not matter?
5. What does Pomeranz think are the crucial factors that enabled
Europe to have an industrial revolution?
Note: You can learn about Pomeranz’s ideas from Marks, pp
104-118.(Already posted it on file)
required that all goods be transported in their ships, and forced
European
New World colonists to trade only with the mother country,
even if
smuggling made such a policy somewhat porous. Mercantilist
ideas also
led to policies that states should use their own raw materials to
manufacture within their own borders anything that was
imported, an
action we saw the English take in the early 1700s to keep Indian
cotton
textiles out. Although mercantilist policies did indeed lead to
the
establishment of industries in European states, industrialization
itself was
not the object: keeping gold and silver from flowing out of the
state and
enriching others was. European states were obsessed with their
silver
stocks: ‘‘the more silver, the stronger the state’’ was how a
German once
put it.40
In these inter-European wars, the fates and fortunes of various
states
rose and fell. As we have already seen, by the end of the
sixteenth century,
Spain’s power had begun to wane, and Portugal proved to be too
small to
mount much of a challenge to the French (or Spanish) in
Europe, or to the
Dutch in Asian waters. The Dutch, being among the first
Europeans to
apply vast amounts of capital to their trading enterprises in both
Asia and
the Americas, saw their fortunes peak in the seventeenth
century, just as
the French and the British were gaining power. Ultimately,
though, the
Dutch did not have the manpower to build a standing army
sufficiently
large to counter the French, and they ultimately allied with the
British to
offset French power on the continent. By the eighteenth century,
Britain
and France had emerged from the seventeenth-century crisis as
the two
most powerful and competitive European states. (See map 3.1.)
The Seven Years’ War, 1756–1763
As the strongest and most successful European states, England
and France
competed not just in Europe but in the Americas and Asia as
well. In the
‘‘long’’ eighteenth century from 1689 to 1815, Britain and
France fought
five wars, only one of which Britain did not initiate. Their
engagement
(with others) in the War of Spanish Succession was ended by
the 1713
Treaty of Utrecht, which established the principle of the
‘‘balance of
power’’ in Europe, that is, that no country should be allowed to
dominate
the others. However, periodic wars between the British and
French
continued.
104
But the most significant was the Seven Years’ War of 1756–
1763, or
what Americans call the French and Indian War and interpret in
terms of
its impact on the American War of Independence of 1776–1783
against
Britain. To be sure, the spark that led to war between Britain
and France
came in the American colonies, and it was in fact the twenty-
two-year-old
George Washington who lit it.41 But it became a global
engagement—
perhaps the first real world war—with British and French troops
fighting
in the backwoods of the American colonies, in Canada, in
Africa, in India,
and in Europe. The outcome was disastrous for the French: they
lost their
colonial claims in both North America (the British got Canada)
and in
India, leading to greater British power and position in both
parts of the
world.42
By 1775, therefore, the processes of state building in Europe
had led to
the creation of a system defined by war, which favored a
particular kind of
state exemplified by the ones built in Britain and France.
Balance of power
among sovereign states, not a unified empire, had become the
established
principle, and Britain had emerged as the strongest European
state. But
that does not mean that it was the strongest or richest state in
the world—
far from it. To be sure, Mughal power in India was declining in
the early
1700s, and as we will see in the next chapter, the British were
able to
begin building a colonial empire there. But the British were still
too weak
to be able to contest China’s definition of the rules of trade in
Asia. When
they tried, most famously in 1793 under Lord Macartney’s
mission, the
Chinese emperor sent them home with a stinging rebuke, and
the British
could do nothing about it. However, the British Isles were
fortunate
enough to be the location for the start of the Industrial
Revolution, which
105
was gaining steam even as Lord Macartney was sailing back to
London.
And when the British learned to apply the tools of the Industrial
Revolution to war, the global balance of power between Britain
and China
tipped. That is the story of the next chapter.
106
CHAPTER FOUR
The Industrial Revolution and Its
Consequences, 1750–1850
In 1750, nearly all of the world’s 750 million people, regardless
of where
they were or what political or economic system they had, lived
and died
within the biological old regime. The necessities of life—food,
clothing,
shelter, and fuel for heating and cooking—mostly came from the
land,
from what could be captured from annual energy flows from the
sun to
Earth.1 Industries too, such as textiles, leather, and
construction, depended
on products from agriculture or the forest. Even iron and steel
making in
the biological old regime, for instance, relied upon charcoal
made from
wood. The biological old regime thus set limits not just on the
size of the
human population but on the productivity of the economy as
well.
These limits would begin to be lifted over the century from
1750 to
1850, when some people increasingly used coal to produce heat
and then
captured that heat to fuel repetitive motion with steam-powered
machines,
doing work that previously had been done with muscle. The use
of coal-
fired steam to power machines was a major breakthrough,
launching
human society out of the biological old regime and into a new
one no
longer limited by annual solar energy flows. Coal is stored solar
energy,
laid down hundreds of millions of years ago. Its use in steam
engines freed
human society from the limits imposed by the biological old
regime,
enabling the productive powers and numbers of humans to grow
exponentially. The replacement—with steam generated by
burning coal—
107
of wind, water, and animals for powering industrial machines
constitutes
the beginning of the Industrial Revolution2 and ranks with the
much earlier
agricultural revolution in importance for the course of history.
The use of
fossil fuels—first coal and then petroleum—not only
transformed
economies around the world but also added greenhouse gases to
Earth’s
atmosphere. How and why this major transformation happened,
and what
consequences it had, thus are vitally important matters in world
history and
will be the focus of this and the following two chapters.
To understand the Industrial Revolution, we will use once again
the tool
of conjuncture, that is, the coming together at a particular point
in time of
otherwise separate historical developments and processes. In the
case of
the Industrial Revolution, the conjuncture involves the playing
out around
the world of growth potential in the biological old regime, the
extension of
European state conflicts around the globe, the peculiar nature of
New
World colonies, and the chance location of, and challenges for
operating,
coal mines in England. In particular, I will consider the ways
cotton
textiles and the British need for coal contributed to the
Industrial
Revolution.
Cotton Textiles
The Industrial Revolution is commonly thought to have begun
in
eighteenth-century England with the mechanization of the
process for
spinning and weaving cotton thread and cloth. The spinning
jenny, the
water frame, and the ‘‘mule’’ all have been taken as evidence of
English
inventiveness and hence contribute to a Eurocentric story line of
the rise of
the West. The problem is, while it is true that England was the
first place
to revolutionize cotton manufacture by using steam-powered
machinery,
how and why it happened can only be understood in a global
context.3
In the late seventeenth century, the English developed a strong
desire
for the Indian cotton textiles commonly known as calicoes. As
one man
observed: ‘‘On a sudden we saw all our women, rich and poor,
cloath’d in
Callico, printed and painted; the gayer the better.’’ Another
complained:
‘‘It crept into our houses, our closets and bedchambers;
curtains, cushions,
chairs, and at last beds themselves were nothing but Callicoes
or Indian
stuffs. In short, almost everything that used to be made of wool
or silk,
relating either to dress of the women or the furniture of our
houses, was
supplied by the Indian trade.’’4 These observations by
contemporaries
around 1700 raise some interesting questions: Why were the
English
importing so much Indian cotton? How did it get there? How
did they then
create and industrialize a cotton textile industry?
108
The reason the English imported so much Indian cotton around
1700 is
it was of high quality and lower price than England’s
domestically
produced textiles (in particular linen and wool). It felt good
next to the
skin, it was lightweight for summer wear, it could accept bright
dyes for
color, and most of all, it was less expensive than anything the
English
could manufacture themselves. Indeed, India around 1700 was
the largest
exporter of cotton textiles in the world and supplied textiles not
just to
meet English demand, but throughout the world as well.
Southeast Asia,
East and West Africa, the Middle East, and Europe were major
export
markets, in addition to the large domestic Indian market. No
wonder that
the demand for Indian cotton in the eighteenth century was
‘‘greater than
all the weavers in the country can manufacture,’’ and that India
accounted
for fully one-quarter of the world manufacturing output in
1750.5
Like so many things desired by Europeans and supplied by
Asians—at
first luxury items for the elite such as silk or porcelain, but
increasingly
products like tea from China for a mass market6 —cotton
textiles were
produced well and cheaply in India. The British textile
manufacturers
focused on the ‘‘cheap’’ part and complained that with
relatively higher
wages, British manufacturers could not compete. India had a
competitive
advantage in the eighteenth century, being able to undersell in
the world
market virtually any other producer of textiles. Some thought
the reason
for cheap Indian textiles was a low living standard, or a large
population
earning depressed wages, but all of those have been shown to
not be true:
Indian textile workers in the eighteenth century had just as high
a standard
of living as British workers.7 So, if it was not a low standard of
living that
gave India its competitive advantage, what did?
In a word: agriculture. Indian agriculture was so productive that
the
amount of food produced, and hence its cost, was significantly
lower than
in Europe. In the preindustrial age, when working families spent
60–80
percent of their earnings on food, the cost of food was the
primary
determinant of their real wages (i.e., how much a pound, a
dollar, a real, or
a pagoda could buy). In India (and China and Japan as well), the
amount of
grain harvested from a given amount of seed was in the ratio of
20:1 (e.g.,
twenty bushels of rice harvested for every one planted), whereas
in
England it was at best 8:1. Asian agriculture thus was more than
twice as
efficient as British (and by extension European) agriculture, and
food—the
major component in the cost of living—cost less in Asia. Thus
although
nominal wages may have been lower in India, the purchasing
power—the
real wage—was higher in India.
In the biological old regime, productive agriculture was Asia’s
competitive advantage, even in industry. The causal chain went
like this:
109
high per-acre yields V low-priced food V relatively low wages
V
comparative advantage. In England, the causal chain was like
this: low
per-acre yields V high-priced food V relatively high wages V
comparative
disadvantage. The question then becomes, how did the British
begin to
reverse this comparative disadvantage?
In part, as we saw in the previous chapter, they did it by raising
tariffs
on imports to Britain of Indian textiles, and the outright
banning of the
importation of some kinds of Indian cotton goods—that is,
mercantilist
protectionism. Had the British not done that in the early
eighteenth
century, there is little reason to believe they would have made
much
progress in competing against Indian producers and establishing
much of a
cotton textile industry in the first place.8 But also, the British
had colonies
in the Americas and acquired their ‘‘jewel’’ in India. Both
became
intimately connected with the story of the rise of a cotton
textile industry
in Britain.
India
Indeed, where England had very little by way of overseas
empire in 1650,
it soon began putting one together, preying on Portuguese and
Spanish
possessions in the East and West Indies (i.e., India and the
Caribbean),
competing with the Dutch in both regions of the world, and
battling France
in the eighteenth century. Curiously, though, the agents for this
extension
of European interstate conflict around the world were not at
first the
governments of European states but private trading companies,
the first
being the Dutch Vereenigde Oost-Indische Compagnie (VOC,
East India
Company), the English East India Company (EIC), and the
Compagnie
française des Indes occidentales (French West India Company).
Although each was formed at different times and had slightly
different
organization, all were private companies chartered by their
governments
and given monopoly rights to trade with Asia, all in keeping
with
mercantilist ideas. They also differed from mere trading
expeditions in that
they were formed with a permanent capital and stock that could
be traded
—to that extent, the East India companies are the forerunners of
the
modern corporation, and their success at organizing trade and
raising
profits meant that the corporation would play an increasingly
important
role in European industrialization. But in the seventeenth and
eighteenth
centuries, their purpose was to reap profits from trade with
Asia.
The Dutch VOC, though, seeing itself as an extension of
Protestant
Dutch interests and hence deeply hostile to the Catholic powers
of Spain
and Portugal, saw trade and war as intimately connected. In a
terse 1614
letter to his directors, the Dutch VOC governor-general
observed: ‘‘You
110
gentlemen ought to know from experience that trade in Asia
should be
conducted and maintained under the protection and with the aid
of your
own weapons, and . . . [s]o trade cannot be maintained without
war, nor
war without trade.’’9 The Dutch then effectively pursued this
strategy
throughout the seventeenth century, taking Melaka from the
Portuguese,
seizing Java and making it into a sugar-producing colony, and
trying to
establish a colony on the Chinese island of Taiwan.
The English EIC, by contrast, was more interested in trade and
the
profits of trade than in war, at least at first. In the century after
its founding
in 1600, the directors insisted that ‘‘our business is trade, not
war.’’10 To
avoid conflicts, the English EIC concentrated trade in India,
where Indian
states were weak and European competitors few, especially in
Bengal and
Madras. But by the late seventeenth century, that began to
change as the
French established forts nearby. And when the British and
French warred
in Europe, their forces (however small) clashed in India, with
the French
usually getting the upper hand because they began enhancing
their war-
making capability by enlisting Indians as regulars, known as
Sepoys, into
their army. In the 1750s the British EIC followed suit, and by
the eve of
the Seven Years’ War, each had nearly ten thousand men in
arms—mostly
Indians—on the Indian coast.
In the meantime, the political and military power of the great
Mughal
empire had seriously declined. At its height it was capable of
mobilizing
perhaps a million troops; after the death of its last great leader,
Aurangzeb,
in 1707, the empire declined as regional political and military
leaders
asserted their independence from the Mughals. One of those
leaders, the
nawab of Bengal, took control of the British trading port at
Kolkata
(Calcutta) and demanded increased payments from the EIC for
the
privilege of trading there.
The British resisted, sent a force of some two thousand men
under the
leadership of Robert Clive and, together with other Indian
forces opposed
to Bengal, defeated the nawab’s French-assisted forces at the
Battle of
Plassey in 1757. They captured and executed the nawab, got a
more pliable
replacement, and by 1765 received the right to collect tax
revenue—a huge
sum—from Bengal. In the meantime, of course, the Seven
Years’ War had
begun, and British and French forces had at it up and down the
Indian
coast, with the British winning a decisive victory over the
French at
Pondicherry in 1760. This was the start of the British empire in
India, and
over the next fifty years the extent of British control widened,
with the
entire subcontinent becoming a formal colony in 1857. (See map
3.1.)
The Seven Years’ War—or more precisely, the British victory in
the
Americas and in India—is important to the story of how Britain
became a
111
cotton-textile-producing, rather than -importing, country. Recall
that the
British government had banned the importation of Indian
textiles in 1707
for the purpose of allowing its domestic cotton industry to get
going,
which it did, in the area around the town of Lancaster. But
because of
technical difficulties in copying Indian dyeing techniques and
because of
its higher wages/higher prices, Lancashire (the region around
Lancaster)
produced mostly for the British home market, still being bested
in the
world market by Indian textiles traded by the EIC. For the
British cotton
textile industry to grow, it therefore needed export markets.
And there was
a growing market in the New World because of its peculiar
institutions of
slavery, plantations, and mercantilist trade restrictions.
The New World as a Peculiar Periphery
European New World agriculture from the beginning was export
oriented.
Throughout the Caribbean and South America, mostly all sugar,
tobacco,
and cotton was produced on plantations that used African slaves
because
of labor shortages caused by the Great Dying and the
unwillingness of
Europeans to migrate to the New World. Unlike peasants in
India and
China or serfs in eastern Europe, African slaves in America did
not grow
much of their own food. Food, especially fish and grain, had to
be
imported, mostly from the North American colonies. Slaves also
had to be
clothed, creating a demand for cheap cotton textiles. Additional
quantities
of Indian textiles were traded in West Africa for slaves who
were then sold
in the Caribbean. New World products—sugar, tobacco, raw
cotton—were
taken back to England.11
At each point in the triangular Atlantic trade, the English made
profits
and by colonial legislation tried to ensure that the New World
would
remain producers of raw materials only and consumers of the
industrial
products of Britain. Smuggling or trading with the enemy,
whether Dutch
or French, was pervasive, but by the early eighteenth century,
‘‘colonial
trade conformed in almost every particular to the navigation
system . . .
[and] smuggled goods accounted for a tiny fraction of all
quantities
handled.’’ Of course, the colonists in both the Caribbean and
North
America were Englishmen, and they too looked for ways to
profit from a
system that denied other nationals, especially the Dutch or the
French,
from getting a piece of Britain’s colonial trade.12
This triangular trade and in particular the linkage between the
slave
trade and textiles fueled the growth of British shipping and
established
Lancashire as a center of cotton textile manufacture. Raw cotton
was
imported mostly from the Levant in the Ottoman empire and the
British
colonies in the Caribbean, and by the 1780s it was spun into
thread in
112
newfangled ‘‘factories’’ using water power and employing
hundreds of
workers in one place. As the Lancashire manufacturers became
more
proficient and the prices of their textiles declined, they even
exported them
to Africa, especially whenever Indian textiles were expensive.
The real
boom to British cotton textile production came after American
independence in 1793 when Eli Whitney’s invention of the
cotton gin
made it possible to use short-staple and much cheaper American
cotton.
With another series of innovations derived from the application
of steam
power, as we will see shortly, mechanizing both spinning and
weaving
over the years from 1815 to 1840, the productivity of the
Lancashire
textile factories surged again, resulting in ever lower prices and
the ability
to outcompete Indian textiles in the world market. Indian textile
producers
had created a worldwide market for cotton textiles—the British
then
captured it.13
When that happened, the British became advocates of ‘‘free
trade’’ and
abandoned both mercantilist theory and practice and tariffs on
imports.
Indeed, ‘‘free trade’’ became the ideological mantra of imperial
Britain at
the height of its global power in the nineteenth century.
Mercantilism, at
least as it applied in the Atlantic world, had been dead since the
victory of
the Americans in their War of Independence from Britain. To
the British,
their former American subjects and colonists became
‘‘foreigners, subject
to all the provisions of the Navigation Laws,’’14 which
restricted the
importation of raw cotton, potentially strangling the British
textile industry
and giving rise to calls for ‘‘free trade.’’ Free trade with the
new United
States after 1783 showed the fallacy of the argument that
British
manufactures could grow only with a monopoly on colonial
markets, and
the American South with its cotton plantations worked by
African slaves
and their descendants became the major supplier of raw cotton
to the mills
of Lancashire.
Although this story of the rise to global competitiveness of the
British
cotton textile industry sounds Eurocentric, it really is not, for
British
success was contingent upon a number of worldwide
developments that
were not of their own making. In the first place, the British
were at a
competitive disadvantage to Indian producers and would have
remained so
except for several coincidences. The Glorious Revolution of
1688–1689
brought to power a government willing to use state power to
protect its
domestic manufacturers; and the New World developed as a
peculiar
periphery that, by the accident of the Great Dying and colonial
legislation,
provided a market for British manufactured goods. In the second
place, the
British were fortunate to develop a usable coal-fueled steam
engine, which
further revolutionized cotton textile production, making it even
more
113
productive and its products so cheap that the British could
undersell Indian
textiles not just in Africa but, interestingly, in India as well.
For that part
of the story, we now look at the innovations in coal and steam
engines.
New Sources of Energy and Power
Until about 1830, the story of cotton textiles for the most part
remains one
that unfolds within the biological old regime; that is, everything
about it
depended on the annual flows of solar energy and their capture
by
humans.15 To be sure, the early British ‘‘factories’’ had begun
to use water
power, but there was a limit to how much that could increase
cotton textile
production. Indeed, there is every reason to think that cotton
textile
production would have reached serious limits within the
biological old
regime, leading not to an industrial revolution but to an
economic dead
end, had it not been for coal, the steam engine, and iron and
steel
production, which truly launched the Industrial Revolution and
allowed
Britain to break out of the constraints imposed by the biological
old
regime. To see how and why, we need to take a closer look at
what was
happening to the most advanced biological old regime
economies, starting
with China and then looking at England. What we will see is
that all old
regime economies were beginning to push up against serious
ecological
constraints that would have stopped all of them from developing
an
industrial revolution. Except for a few chance occurrences and a
vast
global conjuncture, we all now might still be living in the
biological old
regime.
China
Two favored explanations of the Industrial Revolution in
Europe have
focused on population dynamics and the growth of free markets.
By
various techniques and practices, mostly late marriage,
European families
were able to keep their sizes smaller than ‘‘naturally’’ possible.
Smaller
family size meant a smaller population overall, leaving greater
surpluses in
the hands of families to invest in improving agricultural and
industrial
productivity. Fewer people working harder to make their
investable
surpluses grow—an ‘‘industrious revolution,’’ it is said—grew
inexorably
into the Industrial Revolution.16
The market-driven story line of industrialization suggests that
the
establishment and growth of markets for commodities, land,
labor, and
capital in Europe enabled European producers to be much more
efficient
and hence to accumulate sufficient capital to invest in
improving
agricultural and industrial productivity. Also necessary for the
success of
114
markets was a state that protected (or at least respected) private
property
rights. This combination likewise, according to the Eurocentric
version of
the origins of the modern world, grew more or less naturally
into the
Industrial Revolution.
Of course, the population- and market-driven story lines of
industrialization are not incompatible, and many historians have
melded
them together in explaining why Europeans were uniquely
capable of
launching an industrial revolution. As proof, they often point to
China as a
counterexample. China, it is alleged, had ‘‘a preindustrial
demographic
regime,’’ in which nothing was done to keep birthrates down.
Hence,
population surged, eating up any surplus above subsistence and
rendering
the investments necessary for an industrial revolution
impossible.17
Similarly, it is alleged, China was ‘‘despotic’’: it had a state
that meddled
in private affairs, property rights were not respected, and
markets could
not operate efficiently. Hence, it is concluded, there was no
possibility for
an industrial revolution.
There is only one thing wrong with these assumptions about
what
‘‘went wrong’’ in China: they are wrong. As I will show below,
Chinese
families in fact had numerous ways—albeit different from
Europeans—of
limiting their size and hence keeping the overall Chinese
population above
subsistence levels. Also, Chinese markets of all kinds not
merely existed,
but arguably functioned better and more efficiently than those
in Europe. If
both of those are true for China, then their value as
‘‘explanations’’ for
why the Industrial Revolution occurred in Europe is
questionable. To see
why, we must take a closer look at China.
As mentioned earlier in this chapter, agriculture in China (as
well as in
India and numerous other parts of Asia) was highly productive,
harvesting
twenty bushels of rice for every one sown. Rice has the unique
capability
of gaining nutrients not directly from the soil, but from the
water (and so it
is grown in ‘‘paddies’’), eliminating the need for the land to lie
fallow, as
was the custom in Europe, to regain its fertility. Additionally,
Chinese
farmers had learned how to prepare the soil, to irrigate, to
fertilize, and to
control insect pests in order to maximize the harvest yield.
Moreover,
farmers in the southern half of China could get two or
sometimes three
harvests per year from the same plot of land, drawing the
amazement of
early-eighteenth-century European travelers to China. ‘‘By what
art can
the earth produce subsistence for such numbers [of people]?’’
asked the
Frenchman Pierre Poivre in the 1720s.
Do the Chinese possess any secret arts of multiplying grain and
provisions necessary for the nourishment of mankind? To solve
my
115
doubts I traversed the fields, I introduced myself among the
laborers, who
are in general easy, polite, and knowledgeable of the world. I
examine,
and pursue them through all their operations, and observe that
their secret
consists simply in manuring the fields judiciously, ploughing
them to a
considerable depth, sowing them in the proper season, turning
to
advantage every inch of ground which can produce the most
considerable
crop, and preferring to every other species of culture that of
grain, as by
far the most important.18
Such an impressively productive agriculture certainly allowed
the
Chinese population to grow, from 140 million in 1650, to 225
million in
1750, and then to 380–400 million by 1850. Numbers like these
also
convinced European observers, in particular Adam Smith and
Thomas
Malthus, whose ideas about markets and population have so
shaped
Eurocentric views of the modern world, that the Chinese just
could not
control their population growth. Malthus believed that
populations like the
Chinese who could not control their growth would overshoot the
capability
of the land to support their numbers until ‘‘negative’’
population checks,
such as famines or wars, reduced the population size. Malthus
also
believed that Europeans avoided those fates by having
‘‘preventative’’
checks on population growth.
Where Malthus certainly was right about Europeans, he was
wrong
about the Chinese. The fact is, they could—and did—control
their family
size, although in ways quite different from the Europeans.
Although
almost all Chinese women married and married early, Chinese
families
developed many methods for controlling the number of children.
Abstention from having sexual relations, especially early in the
marriage,
was a preferred mechanism and was enforced by married
couples living
with their parents. Infanticide, especially of daughters, was
another means
to limit family size, leading as well to a gender-unbalanced
population of
more men than women, and hence of forced celibacy for many
poorer
men. As James Lee and Wang Feng summarize the Chinese
demographic
system:
In contrast to the European system, in which marriage was the
only
volitional check on population growth, the Chinese demographic
system
had multiple conscious checks, and was therefore far more
complex and
calculating than Malthus or his successors thought. As a result .
. .
population never pushed the economy to subsistence levels.19
Nevertheless, because of the productivity of agriculture and the
ability
of the Chinese economy to produce more than enough food for
its
116
population, the population did in fact grow, and as mentioned
above, it
grew rapidly from 1750 to 1850.20 In the densely populated
core areas of
the Pearl River delta in south China, along the southeast coast,
and in the
Yangzi River delta, populations did reach the size where people
started
migrating out into less-populated areas. Sometimes these
regions had
exceptionally fertile soil that could be brought into production
by clearing
the land, as in Hunan up the Yangzi River from Shanghai, or in
the West
River valley in Guangxi province, or sometimes the land that
was brought
into production was more marginal and less fertile, as in the
Jiangxi
highlands on the southern bank of the Yangzi River.21
Wherever new land was being brought into agricultural
production,
especially by 1800 when it was land that was not as fertile or
productive as
land in the densely populated core regions, that was an
indication that the
limits of growth within the biological old regime were being
reached. That
does not mean that a Malthusian disaster was imminent—the
Chinese were
in fact very much in control of their reproductive capabilities—
but that
good agricultural land was becoming in short supply. The
reason for this is
that the four necessities of life—food, clothing, shelter, and
fuel—all came
from the land and hence were in competition. Clearing land for
food
decreased the amount of wood available for fuel, either to cook
and heat
homes or to make charcoal for industrial purposes. Switching
land from
cotton to rice production also put pressure on the supply of the
raw
material for clothing, while doing the opposite would decrease
the amount
of food available. There just was not much room for
maneuvering when
the limits of the biological old regime were being reached, as
they were in
China in the late eighteenth century,22 and, as we will see, in
Britain too.
It was not just that meeting the needs for sustained population
growth
meant increasing pressure on the land and decreases in other
things at the
expense of food, but that to keep food production increasing
while at the
same time keeping supplies available for clothing, shelter, and
fuel meant
that greater and greater amounts of labor and capital had to be
expended in
agriculture just to keep pace. For instance, clearing land was
expensive
and so too was building irrigation works or terracing fields from
hills, all
of which improved the output of Chinese agriculture in the
eighteenth and
nineteenth centuries. Allocating more labor also could increase
output, and
Chinese farming families did that too: planting rice in nurseries
and then
transplanting it to the fields or picking insects off rice plants by
hand, for
instance, also increased agricultural yields and sustained a
growing
population. So too did capturing and recycling nitrogen from
human and
animal waste; Asian farmers were biological-old-regime
champions in
maintaining or even increasing the fertility of their fields.
117
Markets
Another way the Chinese economy improved both overall
production
levels and productivity was by the use of markets, especially for
agricultural commodities. It used to be thought that markets
were first and
most highly developed in Europe (reading backward from the
Industrial
Revolution to find reasons why it happened there first). But in
the past
thirty years, historians of China have shown how fully
developed and
efficient markets were in eighteenth- and nineteenth-century
China.23
Peasant farmers in the Pearl River and Yangzi River deltas, for
instance,
came to specialize in sericulture (that is, the whole process of
producing
silk), raising silkworms and growing the mulberry trees with
which to feed
the silkworms, boiling the cocoons to obtain the silk threads,
then
spinning, weaving, and dyeing the silk. Other areas might
specialize in
cotton, sugarcane, or other nonfood agricultural crops.
Such specialization meant that those peasant producers had to
obtain
their food from other sources, usually places upriver that came
to
specialize in rice and could easily export it on boats to the more
densely
populated core regions. Massive investments in canals by both
private
parties and the state vastly extended and improved the Chinese
inland
water transportation system, linking China from Tianjin in the
north to
Guangzhou in the south by water. Efficient water transportation
facilitated
the movement of grain throughout the Chinese empire and the
growth of
markets and provided the material foundations for maintaining
some of the
world’s largest cities.
Initially, the Chinese state intervened in the food markets quite
regularly to ensure that peasant producers and urban consumers
alike
would be ensured adequate food supplies,24 but by the mid-
eighteenth
century the Chinese state was increasingly willing to allow
markets and
merchants to handle the movement of grain across huge
distances—up to a
thousand miles—from where it was produced to where it was
consumed.
Measures of the efficiency of these markets show that they were
more
efficient than contemporary markets in France, England, or the
United
States.25 Additionally, Chinese markets for land, labor, and
capital all
functioned well and in some ways more efficiently than
comparable
markets in European countries.26
In short, eighteenth-century China looked as ‘‘developed’’ as
any other
developed part of the world, whether measured by levels of
agricultural
productivity, sophistication of manufactures and markets, or
levels of
consumption. Chinese families regulated their size and were
responsive to
changing economic opportunities, limiting their size when those
opportunities diminished in order to maintain consumption
above
118
subsistence levels; specialization of function gave rise to
markets and a
highly commercialized economy; and an extensive water-based
transportation system allowed the efficient movement of goods
and people
throughout the empire.
Yet China’s highly developed market economy did not lead to
an
industrial breakthrough. Instead, by the nineteenth century,
there were
plenty of indicators that China was pushing up against
ecological
constraints imposed by the biological old regime. In several
areas, fuel
became in short supply in the early 1800s, with peasant families
turning to
rice straw and chaff for heating and cooking rather than wood.
Moreover,
some market exchanges between densely populated core regions
and
developing peripheral areas also served to slow Chinese
economic growth.
One of the advantages of markets and a good transportation
network is
that they allow some areas to specialize in what their natural
resources
make most profitable and to exchange that produce with others,
enabling
both to be more productive and allowing everyone’s income to
rise. At
least that is the theory, and to a point that is the way markets
functioned in
China. However, the exchanges began to break down regarding
the trade
of raw cotton from cotton-producing regions in return for
manufactured
goods, cotton textiles in particular, from the highly developed
core regions
in the lower Yangzi and Pearl River deltas.
Throughout China, rural families were free to decide what and
how
much to grow and how to allocate family labor on the farm. To
this extent,
they differed markedly from African slaves in the New World or
serfs in
eastern Europe, both of whom had their freedom curtailed and
production
decisions made by their owners or overseers. Thus those
Chinese peasants
who migrated to more peripheral areas, like their counterparts
in the more
developed cores, were free to make their own decisions.
Increasingly, what
they decided was that it was in their interest to spin and weave
their own
cotton textiles for their own use and for local exchange, rather
than
concentrating on rice or raw cotton and importing the finished
goods. In
effect, large parts of rural China underwent a process of
‘‘import
substitution,’’ producing their own textiles. Not only did they
reduce the
amount of raw cotton sold to the textile-producing centers, but
they also
increased the area given over to cotton and hence decreased the
amount of
rice they were willing to export as well.27
The freedom of Chinese peasant families thus may have spurred
what
might be called ‘‘self-sufficient proto-industrialization’’ in
peripheral
areas, but that acted as a constraint on the growth of an
industrial cotton
textile industry in China’s core regions. Contributing to the
willingness of
Chinese peasant families in peripheral areas to spin and weave
their own
119
textiles may have been the long-standing norm that ‘‘men plow,
women
weave.’’ It was not just that ‘‘women weave,’’ but that they
weave in the
household. Chinese families thus placed a high value on
mothers and
daughters staying at home to do the weaving, rather than
leaving home to
work in a factory, as English and Japanese girls did.28
Paradoxically, the
freedom of peasant farmers throughout China’s core and
peripheral
regions, when compared with the limited freedoms of slaves and
serfs in
the European system, constrained China’s ability to continue
developing a
textile industry in its most highly developed core regions.
In summary, China had a highly developed market economy
within the
constraints of the biological old regime. Nonetheless, that
regime placed
ecological limits upon growth, and the freedom of Chinese
peasants
coupled with practices governing the sexual division of labor,
all
combined, meant that China was bumping up against the limits
of growth
by the mid-1800s. Food, clothing, shelter, and fuel competed for
land, and
to get more from the land, the Chinese lavished increasing
amounts of
labor on agriculture. The dynamics of specialization, increased
market
exchanges, and improved transportation in the context of the
biological old
regime and the particularities of China’s situation were pushing
it toward
an increasingly labor-intensive agriculture and the depletion of
land
resources, rather than toward an industrial revolution. As fuel
supplies
from wood declined, the Chinese turned again, where they
could, to coal.
Coal was available and used in North China, parts of Central
China, and in
South China too, where it was used in the iron foundries of
Foshan town
near the major city of Guangzhou. Nevertheless, this
availability of coal in
China did not lead to an industrial revolution there, or
elsewhere in Eurasia
for that matter. Particular circumstances came together in one
place,
England, that did create the conditions for the leap into the
fossil-fueled
industrial age that became the modern world.
Exhausting the Earth29
Indeed, over the period from 1400 to 1800, the dynamics of the
biological
old regime, especially in those areas most densely populated
and even with
the institutions of private property, markets, and effective
states, were
leading not to a breakthrough to a ‘‘modern world’’ of fossil-
fuel energy
and electrified industry and homes, but to an increasingly
intensive using
up of resources. In part, this outcome was driven by a
significant
worldwide population increase, rising from 380 to 950 million
people,
most of that coming in the century following the depths of the
seventeenth-
century global crisis, which drove the removal of forests for
more
farmland (and food for people).30 Interestingly, climatologists
now think
120
Why Europe and the West? Why Not
China?
David S. Landes
T he world history of technology is the story of a long,
protracted inversion.As late as the end of the first millennium
of our era, the civilizations ofAsia were well ahead of Europe in
wealth and knowledge. The Europe of
what we call the Middle Ages (say, tenth century) had regressed
from the power
and pomp of Greece and Rome, had lost much of the science it
had once possessed,
had seen its economy retreat into generalized autarky. It traded
little with other
societies, for it had little surplus to sell, and insofar as it
wanted goods from outside,
it paid for them largely with human beings. Nothing testifies
better to deep poverty
than the export of slaves or the persistent exodus of job-hungry
migrants.
Five hundred years later, the tables had turned. I like to
summarize the change
in one tell-tale event: the Portuguese penetration into the Indian
Ocean led by
Vasco da Gama in 1498. This was an extraordinary
achievement. Some scholars will
tell you that it was some kind of accident; that it could just as
easily have been
Muslim sailors, or Indian, or Chinese to make the connection
from the other
direction. Did not the Chinese send a series of large fleets
sailing west as far as the
east African coast in the early fifteenth century—bigger, better
and earlier than
anything the Portuguese had to show?
Don’t you believe it. These affirmations of Asian priority are
especially prom-
inent and urgent nowadays because a new inversion is bringing
Asia to the fore. A
“multicultural” world history finds it hard to live with a
eurocentric story of
achievement and transformation. So a new would-be (politically
correct) orthodoxy
would have us believe that a sequence of contingent events
(gains by Portugal and
then others in the Indian Ocean, followed by conquests by Spain
and then others
in the New World) gave Europe what began as a small edge and
was then worked
up into centuries of dominion and exploitation. A gloss on this
myth contends that
y David S. Landes is Emeritus Professor of Economics, Harvard
University, Cambridge,
Massachusetts.
Journal of Economic Perspectives—Volume 20, Number 2—
Spring 2006—Pages 3–22
a number of non-European societies were themselves on the
edge of a technolog-
ical and scientific breakthrough; that in effect, European
tyranny (to paraphrase
Thomas Gray’s “Elegy Written in a Country Churchyard”),
“froze the genial current
of the [Asian] soul.”
A variant on this history-as-accident (or luck) is the pendulum
approach
associated with Jack Goody’s (1996) book, East in the West.
Everything starts on an
even keel thanks to the allegedly common heritage of the
Bronze Age; but then
different parts move ahead, only to be caught up and passed by
others, which then
lose ground to their predecessors. So Europe was just especially
lucky, taking the
lead at the crucial turn to the Industrial Revolution. But Asia’s
turn will now come;
indeed is already coming. As Goody (pp. 231–232) writes: “[I ]t
is a pendular
movement that continues today, with the East now beginning to
dominate the West
in matters of the economy.” As for efforts to understand this
European success—
especially explanations based on allegedly deep characteristics
that were present in
Europe but wanting in China—such efforts are irrelevant, writes
Goody (p. 238):
. . . since all these features must have been present [in China] at
the earlier
period. Those discussions can be seen for what they are, as
representing the
understandable but distorting tendency of Europeans to inflate
their overall
contribution to world society and even to ‘Western civilisation’,
a tendency
reinforced by their undoubted achievements over the past few
centuries. Such
inflation of oneself inevitably involves the deflation of others;
self-congratulation
is a zero-sum game.
But of course, Westerners were not alone in noticing some
European deep
characteristics. Thus Abu Talib, an Indian Muslim visitor to
Britain late eighteenth
century, commenting on British precocity in mechanization:
“The British,” he
wrote (cited in Khan, 1998, p. 303), “were endowed with a
natural passion for
technical innovation. They possessed inventive skills and
preferred to perform even
minor routine jobs with the aid of mechanical instruments rather
than manually.
They had such great passion for the use of technical instruments
that they would
not perform certain tasks unless the necessary instruments were
at their disposal.”
The French, he went on, were not like that.1
I shall return later to this revisionist debate. Here, suffice to
say: 1) The
Portuguese success was the result of decades of rational
exploration and extension
of navigational possibilities in an ocean (the south Atlantic)
that was hostile to
traditional techniques of navigation, which essentially involved
following the coast-
line. This technological enhancement rested in turn on a
systematic utilization of
astronomical observations and calculations, taken from the
Muslims and transmit-
ted largely by Jewish intermediaries, which allowed the
Portuguese to follow winds
and currents across the south Atlantic, and then use a
knowledge of latitude to
swing back around the tip of Africa and into the Indian Ocean.
2) The Chinese
1 Khan (1998, p. 328, n. 122) notes further that the Arabic
lacked the vocabulary needed to speak of
factory manufacture or machinery. For the latter, Abu Talib
used “wheels and tools.”
4 Journal of Economic Perspectives
abandonment of westward exploration was partly the result of
contingent political
events; but at bottom it reflected the values and structures of
Chinese society and
civilization. 3) European exploitation of the breakthrough rested
on a disparity of
power technology (better powder and better guns) as well as on
navigational
superiority.
The extension of European power into other parts of the world
was the
expression of these and other disparities. Why other regions did
not keep up with
Europe is an important historical question, for one learns almost
as much from
failure as from success. It is not possible in brief compass, of
course, to pose this
question for every non-European society or civilization; but
three do deserve
serious reflection: Islam, China, and India. I shall focus in this
essay on China.
The First Chance: Science without Development
The one civilization that was in a position to match and even
anticipate the
European achievement was China. China had two chances: first,
to generate a
continuing, self-sustaining process of scientific and
technological advance on the
basis of its indigenous traditions and achievements; and second,
to learn from
European science and technology once the foreign “barbarians”
entered the Chi-
nese domain in the sixteenth century. China failed both times.
The first failure has elicited much scholarly inquiry and
analysis. And yet it
remains an abiding mystery. The China specialists tell us, for
example, that in a
number of areas of industrial technique, China long anticipated
Europe: in textiles,
where the Chinese had a power-driven spinning machine in the
thirteenth century,
some 500 years before the England of the Industrial Revolution
knew water frames
and mules; or in iron manufacture, where the Chinese early
learned to use coal and
probably coke (as against charcoal) in blast furnaces for
smelting iron and were
turning out perhaps as many as 125,000 tons of pig iron by the
later eleventh
century—a figure not achieved by Britain until 700 years later
(Elvin, 1973, p. 85).2
In general, one can establish a long list of instances of Chinese
priority: the
wheelbarrow, the stirrup, the rigid horse collar (to prevent
choking), the compass,
paper, printing, gunpowder, porcelain. (But not the horse-shoe,
which implies that
the Chinese did not make use of the horse for transport.)
The mystery lies in the failure of China to realize the potential
of some of the
most important of these inventions. One generally assumes that
knowledge and
know-how are cumulative and that a superior technique, once
known, will domi-
2 Elvin (1973) gives the figure as “between 35,000 to 40,000
tons and 125,000 tons,” but says he prefers
the higher estimate. He relies here on Yoshida Mitsukuni, a
Japanese specialist writing in 1967. Work by
Hartwell (1966, p. 34), also advances the higher figure. In Hall
(1985, p. 46), this becomes “at least
125,000 tons.” In this regard, Elvin (p. 285) quotes a
description by Yen Ju-yu of iron works on the
Hupei/Shensi/Szechwan borders with blast furnaces 18 feet
high, using charcoal and hand-operated
bellows (more than ten persons relaying one another) and
working continuously. The iron was
apparently used for castings, and there is no indication of
further refining as either wrought iron or
steel.
David S. Landes 5
nate older methods and remain in use. But Chinese industrial
history offers a
number of examples of technological regression and oblivion.
The machine to spin
hemp was never adapted to the manufacture of cotton; cotton
spinning was never
mechanized; and coal/coke smelting was allowed to fall into
disuse, along with the
iron industry. Why, asks Elvin (1973, pp. 297–298)?
It would seem that none of the conventional explanations tells
us in convinc-
ing fashion why technical progress was absent in the Chinese
economy during a
period that was, on the whole, one of prosperity and expansion.
Almost every
element usually regarded by historians as a major contributory
cause to the Indus-
trial Revolution in north-western Europe was also present in
China. There had even
been a revolution in the relations between social classes, at
least in the countryside;
but this had had no important effect on the techniques of
production. Only
Galilean-Newtonian science was missing; but in the short run
this was not impor-
tant. Had the Chinese possessed, or developed, the seventeenth-
century European
mania for tinkering and improving, they could easily have made
an efficient
spinning machine out of the primitive model described by Wang
Chen. A steam
engine would have been more difficult; but it should not have
posed insuperable
difficulties to a people who had been building double-acting
piston flame-throwers
in the Sung dynasty. The crucial point is that nobody tried. In
most fields, agricul-
ture being the chief exception, Chinese technology stopped
progressing well
before the point at which a lack of scientific knowledge had
become a serious
obstacle.
Why indeed? Sinologists have put forward several partial
explanations. Those
that I find most persuasive are the following.
First, China lacked a free market and institutionalized property
rights. The
Chinese state was always stepping in to interfere with private
enterprise—to take
over certain activities, to prohibit and inhibit others, to
manipulate prices, to exact
bribes. At various times the government was motivated by a
desire to reserve labor
to agriculture or to control important resources (salt and iron,
for example); by an
appetite for revenue (the story of the goose that laid the golden
eggs is a leitmotif
of Chinese history); by fear and disapproval of self-enrichment,
except by officials,
giving rise in turn to abundant corruption and rent-seeking; and
by a distaste for
maritime trade, which the Heavenly Kingdom saw as a diversion
from imperial
concerns, as a divisive force and source of income inequality in
the ecumenical
empire, and worse yet, as an invitation to exit. This state
intervention and inter-
ference encountered evasion and resistance; indeed, the very
needs of state com-
pelled a certain tolerance for disobedience. Still, the goal, the
aim, the ideal was the
ineffable stillness of immobility. When in 1368 the new Chinese
emperor inaugu-
rated a native (Ming) dynasty to replace the defeated Mongol
invaders, he as-
cended the throne in Nanjing as the Hongwu (“Vast Martial”)
emperor. Let not the
name deceive the reader: Hongwu’s goal was anything but war.
He wanted rather
to immobilize the realm. People were to stay put and move only
with the permission
of the state—at home and abroad. People who went outside
China without per-
mission were liable to execution on their return. The Ming code
of core laws also
sought to block social mobility, with severe penalties for those
jumping professional
6 Journal of Economic Perspectives
and occupational barriers. In this regard, Timothy Brook (1998,
p. vii) cites in
epigraph one of the Hongwu emperor’s favorite moral dicta:
Let the state be small and the people few;
So that the people . . . fearing death, will be reluctant to move
great
distances
And, even if they have boats and carts, will not use them.
So that the people . . . will find their food sweet and their
clothes
beautiful,
Will be content with where they live and happy in their
customs.
Though adjoining states be within sight of one another and
cocks crow-
ing and dogs barking in one be heard in the next,
Yet the people of one state will grow old and die without having
had any
dealings with those of another.
These matters reached a wretched climax under the Ming
dynasty (1368–1644),
when the state attempted to prohibit all trade overseas.3 Such
interdictions led of
course to evasion and smuggling, with concomitant corruption
(protection
money), searches for contraband, confiscations and punishment.
All of this neces-
sarily acted to strangle initiative, to increase risk and the cost
of transactions, and
to chase talent from commerce and industry.
A second reason why China did not realize the economic
potential of its
scientific expertise involved the larger values of the society.
The great Hungarian-
German-French sinologist, Etienne Balazs (1968 [1988]; see
also Balazs, 1964), saw
China’s abortive technology as part of a larger pattern of
totalitarian control. He
recognizes the absence of freedom, along with the weight of
custom and consensus
and what passed for higher wisdom. His analysis (pp. 22–23) is
worth repeating:
. . . if one understands by totalitarianism the complete hold of
the State and
its executive organs and functionaries over all the activities of
social life,
without exception, Chinese society was highly totalitarian. . . .
No private
initiative, no expression of public life that can escape official
control. There
is to begin with a whole array of state monopolies, which
comprise the great
consumption staples: salt, iron, tea, alcohol, foreign trade.
There is a monop-
oly of education, jealously guarded. There is practically a
monopoly of letters
(I was about to say, of the press): anything written unofficially,
that escapes
the censorship, has little hope of reaching the public. But the
reach of the
3 The imperial authorities vacillated in their attitude to foreign
trade, now favoring it, now clamping
down; and these tergiversations were in themselves a deterrent
to stable enterprise and capital accu-
mulation. In addition, even when the state relented, it did so in
circumstances that pushed the traders
into illicit operations. Thus, the early Mongol (Yuan) dynasty
(1280–1368) allowed freedom of enter-
prise, but then succumbed to the temptation of instituting a
licensing system. This enabled officials to
play the role of capitalist, financing venturers and dividing
profits 70-30: 70 for the official, 30 for the
working trader. That was greedy, compared to the typical
European 50-50 split. The traders presumably
sought to conceal gains, but in the long run, trade had to suffer.
Why Europe and the West? Why Not China? 7
Moloch-State, the omnipotence of the bureaucracy, goes much
farther. There
are clothing regulations, a regulation of public and private
construction
(dimensions of houses); the colors one wears, the music one
hears, the
festivals—all are regulated. There are rules for birth and rules
for death; the
providential State watches minutely over every step of its
subjects, from cradle
to grave. It is a regime of paper work and harassment, endless
paper work and
endless harassment.
The ingenuity and inventiveness of the Chinese, which have
given so
much to mankind—silk, tea, porcelain, paper, printing, and
more—would no
doubt have enriched China further and probably brought it to
the threshold
of modern industry, had it not been for this stifling state
control. It is the State
that kills technological progress in China. Not only in the sense
that it nips in
the bud anything that goes against or seems to go against its
interests, but also
by the customs implanted inexorably by the raison d’Etat. The
atmosphere of
routine, of traditionalism, and of immobility, which makes any
innovation
suspect, any initiative that is not commanded and sanctioned in
advance, is
unfavorable to the spirit of free inquiry.
In short, to go back to Elvin (1973), the reason the Chinese did
not develop
based on their scientific knowledge is that no one was trying.
Why try? Especially
since the Chinese were not without their own quiet resources to
thwart bureaucratic
interferences and frustrations—reliance on personal and familial
collaboration, for
example, in place of arbitrary or institutional practice in
business. In such matters,
personal trust could yield more dependable performance than
legal rules.
In all this, the contrast with Europe was marked. Where
fragmentation and
national rivalries compelled European rulers to pay heed to their
subjects, to
recognize their rights and cultivate the sources of wealth, the
rulers of China had
a free hand. Again Elvin (1973, pp. 224–225) captures some of
this:
. . . it was the great size of the Chinese Empire which made the
adoption of
the policies of the Ming emperors possible. In a Chinese
subcontinent made
up of smaller independent states, like those of the Five
Dynasties [907-960
C.E.] or the Ten Kingdoms, no government could have afforded
to close itself
off. International economic interdependence (as that between
regions would
have become) would have removed this option; and the need for
diplomatic
and military alliances, and revenue from foreign trade, would
have made
isolationism undesirable. With smaller states, there might also
have been, as
there was in north-western Europe in early modern times, a
closer conscious
identification of the governed with their countries and rulers.
Prior to mod-
ern communications, the immensity of the empire precluded
nationalism.
Whatever the mix of factors, the result seems to have been a
curious pattern of
isolated initiatives and sisyphean discontinuities—up, up, up
and then down
again—almost as though the society were constrained by a
homeostatic braking
mechanism or held down by a silk ceiling. The result, if not the
aim, was a kind of
8 Journal of Economic Perspectives
change-in-immobility; or maybe immobility-in-change.
Innovation was allowed to
go (was able to go) so far and no farther.4
The Europeans knew much less of these interferences. Instead,
they entered
during these centuries into an exciting world of innovation and
emulation that
challenged and tempted vested interests and kept the forces of
conservatism
scrambling. Changes were cumulative, news of novelty spread
fast and a new sense
of progress and achievement replaced an older, effete reverence
for authority. This
intoxicating sense of freedom touched (infected) all domains.
These were years of
heresies in the church, of popular initiatives that, we can see
now, anticipated the
rupture of the Reformation; of new forms of expression and
collective action that
challenged the older organization of society and posed a threat
to other polities; of
new ways of doing and making things that made newness a
virtue and a source of
delight.
Important in all this was the role of the Christian church in
Europe as
custodian of knowledge and school for technicians. One might
have expected
otherwise: that organized spirituality, with its emphasis on
prayer and contempla-
tion, would have had little interest in technology; and that with
its view of labor as
penalty for original sin, it would have had no concern to save
labor. And yet
everything seems to have worked in the opposite direction: The
desire to free
clerics from time-consuming earthly tasks led to the
introduction and diffusion of
power machinery and, beginning with the Cistercians in the
twelfth century, to the
hiring of lay brothers (conversi) to do the dirty work, which led
in turn to an
awareness of and attention to time and productivity. All of this
gave rise on
monastic estates to remarkable assemblages of powered
machinery—complex se-
quences designed to make the most of the water power available
and distribute it
through a series of industrial operations. A description of the
abbey of Clairvaux in
the mid-twelfth century (cited in White, 1978, p. 245–246)
exults in this versatility:
“coquendis, cribrandis, vertendis, terendis, rigandis, lavandis,
molendis, molliendis, suum
sine contradictione praestans obsequium.” The author, clearly
proud of these achieve-
ments, further tells his readers that he will take the liberty of
joking (the medieval
clerical equivalent of, “if you’ll pardon the expression”): the
fulling hammers, he
says, seem to have dispensed the fullers of the penalty for their
sins; and he thanks
God that such devices can mitigate the oppressive labor of men
and spare the backs
of their horses.
Why this peculiarly European joy in discovery? This pleasure in
the new and
better? This cultivation of invention—or what some have called
“the invention of
invention”? Different scholars have suggested a variety of
reasons, typically related
to religious values. One possible reason grows from the Judaeo-
Christian respect for
manual labor, summed up in a number of biblical injunctions.
One example will
suffice: when God warns Noah of the coming flood and tells
him he will be saved,
it is not God who saves him. “Build thee an ark of gopher
wood,” says the Lord, and
4 For example, Max Weber (1922 [1951], as cited in Hall, 1985,
p. 41) argued that the administrative
bureaucracy was undermanned, so that government came to
know and respond to changes only after
they had gotten under way. Hence a pattern of “intermittent and
jerky” homeostatic interventions.
David S. Landes 9
Noah builds an ark to divine specifications. A second and
related reason is the
Judaeo-Christian subordination of nature to man. This belief is
a sharp departure
from widespread animistic beliefs and practices that saw
something of the divine in
every tree and stream (hence the naiads and dryads). Ecologists
today might say
these animistic beliefs were preferable to what was put in their
place, but no one
was listening to pagan nature-worshipers in Christian Europe. A
third reason stems
from the Judaeo-Christian sense of linear time. Other societies
thought of time as
cyclical, returning to earlier stages and starting over again.
Linear time can be
thought of as progressive or regressive, as moving on to better
things or declining
from some earlier, happier state. For Europeans in our period,
the progressive view
prevailed.
In the last analysis, however, I would stress the role of the
market: the fact that
enterprise was free in Europe, that innovation worked and paid,
that rulers and
vested interests were narrowly constrained in what they could
do to prevent or
discourage innovation. Success bred imitation and emulation;
also a sense of power
that would in the long run raise men almost to the level of gods.
The old legends
remained—the expulsion from the Garden, Icarus who flew too
high, Prometheus
in chains—to warn against hubris. The very notion of hubris—
cosmic insolence—is
testimony to some men’s pretensions and the efforts of others to
curb them. But the
doers were not paying attention.
The Second Chance: Learning from the Barbarians
At the time the first Europeans arrived in the Indian Ocean and
made their
way to China, the Celestial Empire as it was called was, at least
in its own eyes, the
premier political entity in the world—first in size and
population, first in age and
experience, untouchable in its cultural achievement, apparently
imperturbable in
its sense of moral and spiritual superiority.5 The Chinese lived,
as they thought, at
the center of the universe; around them, lesser breeds basked in
their glow,
reached out to them for light, gained stature by doing obeisance
and offering
tribute. Their emperor was the “Son of Heaven,” the unique,
godlike representative
of celestial power. Those few who entered his presence showed
their awe by
kowtowing—kneeling and touching their head nine times to the
ground; others
kowtowed to anything emanating from him—a letter, a single
handwritten ideo-
graph. The paper he wrote on, the clothes he wore, everything
he touched partook
of his divine essence. Western diplomats allowed the Chinese to
compel them to
these gestures, which they “considered an essential part of a
tributary system of
foreign relations” (Spence, 1998, p. 42). By doing this, “the
Westerners were
5 These Portuguese sailors of the sixteenth century were of
course not the first Europeans to make their
way to China. The best known of the earlier visitors is Marco
Polo, who came in the thirteenth century
from Venice, then the richest city in Europe, yet thought it a
small town by comparison with what he saw
in Cathay.
10 Journal of Economic Perspectives
unwittingly shoring up the Qing court’s views of China’s
superiority” (Spence citing
Wills, 1984).
Those who represented the emperor and administered for him
were chosen
on the basis of competitive examinations in Confucian letters
and morals. These
mandarin officials were in effect the embodiment of the higher
Chinese culture,
invested with its prestige, imbued with its wholeness and
sublime superiority. Their
self-esteem and haughtiness had ample room for expression and
exercise on their
inferiors and were matched only by their “stunned
submissiveness” and self-
abasement to superiors (Welsh, 1993, p. 16, who in this case
quotes without
reference). Nothing conveyed so well their rivalry in humility
than the morning
audience, when hundreds of courtiers gathered from midnight
on and stood about
in the open air, in rain and cold and fair, to wait for the
emperor’s arrival and
perform their obeisance. They were not wasting time; their time
was the emperor’s.
They could not afford to be late, and punctuality was not
enough: unpunctual
earliness was proof of zeal (Landes, 1983; see also Huang,
1981).
Such cultural triumphalism combined with petty downward
tyranny made
China a singularly bad learner. What was there to learn? This
rejection of the
strange and foreign was the more anxious for the very force of
the arrogance that
justified it. For that is the paradox of the superiority complex: it
is an expression of
insecurity. It is intrinsically brittle; those who nourish it, need
it, and depend on it
are also those who fear nothing so much as contradiction. The
French today are so
persuaded of the superiority of their language that they dither
and tremble at the
prospect of a borrowed word, especially if it comes from
English. The same holds
for Ming China: they were so convinced of their ascendancy
that they quaked
before the challenge of Western technology, which was there for
the learning.
The irony is that those first Portuguese visitors and Catholic
missionaries used
the wonders of western technology to charm their way into
China. The mechanical
clock was the key that unlocked the gates. The mechanical clock
was a European
mega-invention of the late thirteenth century, crucial not only
for its contribution
to temporal discipline and productivity, but its susceptibility of
improvement and its
role at the frontier of instrumentation and mechanical technique.
The water clock
is a dunce by comparison. For the Chinese in the sixteenth
century, the mechanical
clock came as a wondrous machine capable not only of keeping
time but of
amusing and entertaining. Some clocks played music; others
were automata with
figurines that moved rhythmically at intervals. Clocks, then,
were the sort of thing
that the emperor would want to see, that had to be shown him if
only to earn his
favor, that a zealous courtier had to show him before someone
else did. But that was
not so easy. This magical device had to be accompanied. Where
all Chinese
instincts and practice dictated that foreigners should be kept at
a distance, confined
to some peripheral point like Macao and allowed to proceed to
the center only by
exception, the clock, in its sixteenth-century avatar, needed its
attendant clock-
maker and keepers.
The Chinese loved clocks and watches. They were less happy,
though, with
their European attendants. The problem here was the Chinese
sense of the whole-
ness of culture, the link between things, people and the divine.
The Catholic priests
Why Europe and the West? Why Not China? 11
who first brought them these wonderful machines were salesmen
of a special kind.
They sought to convert the Chinese to the one true God, the
trinitarian God of the
Roman church, and the clocks were not only an entry ticket but
an argument for
the superiority of the Christian religion. Were not those who
could make these
things, who possessed all kinds of special astronomical and
geographical knowledge
to the bargain, were they not superior in the largest moral
sense? Was not their faith
truer, wiser? The Jesuits were prepared to make such an
argument, stretching the
while the rules and rites of the Church to fit the premises and
win the sympathy of
an understandably skeptical Chinese elite. (The Chinese
ideographs for ancestor
worship, for example, became the signifiers for the Christian
mass.) But European
laymen made the argument as well. Here is Gottfried Wilhelm
von Leibniz (1646–
1716), mathematician (coinventor of the calculus) and
philosopher (as quoted in
Landes, 1983, p. 45, from a letter written circa 1675):
What will these peoples say [the Persians, the Chinese], when
they see this
marvelous machine that you have made, which represents the
true state of the
heavens at any given time? I believe that they will recognize
that the mind of
man has something of the divine, and that this divinity
communicates itself
especially to Christians. The secret of the heavens, the greatness
of the earth,
and time measurement are the sort of thing I mean.
This argument, whether explicit or implicit, did carry
occasionally. The Cath-
olic missionaries had some small success, although they had
trouble persuading
their open-minded “converts” to be good exclusivists (no other
faith but the “true”
faith) in the European tradition. But most Chinese saw these
pretensions for what
they were: an attack on Chinese claims to moral superiority, an
assault on China’s
self-esteem.
The response, then, had to be a repudiation or depreciation of
Western
science and technology (Cipolla, 1967; Landes, 1983, chapter
2). Here is the K’ang
Hsi emperor, the most open-minded and curious of men in his
pursuit of Western
ways, the most zealous in teaching them (as translated by
Spence, 1974, p. 74):
“[E]ven though some of the Western methods are different from
our own, and may
even be an improvement, there is little about them that is new.
The principles of
mathematics all derive from the Book of Changes, and the
Western methods are
Chinese in origin . . .”
That was the heart-warming myth. So the Chinese, who were not
prepared to
give up clocks, who wanted clocks, who recognized their
Western origin—these
same Chinese trivialized clocks as toys (which for many they
were) or as nonfunc-
tional symbols of status, unaffordable by or inaccessible to
most. Premodern
imperial China did not think of time knowledge as a personal
right. The hour was
sounded by the authorities, and the right to own a timepiece was
a rare privilege.
As a result, although the imperial court set up workshops to
make clocks and got
their Jesuit clockmakers to train some native talent, these
Chinese makers never
arrived at the level of Western horologists—for want of the best
teachers and lack
of commercial competition and emulation. Nor did imperial
China ever develop a
12 Journal of Economic Perspectives
clockmaking trade comparable to that found in European
countries. The same sin
of pride (or indifference) shaped the Chinese response to
European armament.
Here was something that was anything but a toy. Cannons and
muskets were
instruments of death, hence of power, and the Chinese had every
reason to interest
themselves in these artifacts, the more so as the seventeenth
century saw the
progressive dissolution of the Ming dynasty and the conquest of
China by a Tartar
people from the north. These were decades of war, and the
balance of power might
well be tilted by access to these European inventions.
Yet the Chinese never learned to make modern guns. Worse yet,
they had
known and used cannon as early as the thirteenth century but
had forgotten much
of what they had once known. Their city walls and gates had
emplacements for
cannon, but no cannon. Who needed them? The enemies of
China did not have
them. Yet China did have enemies, without and within, and no
European nation
would have been deterred from armament by enemy weakness;
when it came to
death, as in so many other things, the Europeans were
maximizers. European
technology was also monotonic-increasing: each gain was the
basis for further gain.
The Chinese record of advance followed by regression, step-
forward, step-back,
signaled an entirely different process. The Chinese, we are told,
had a proverb: He
who does not go forward will go backward (Peyrefitte, 1992, p.
157). The saying was
apparently as much observation as prescription.6
So it was that in the seventeenth century, when the Portuguese
in Macao
offered three cannon to the emperor by way of gaining favor,
they had to send
three cannoneers along with them. Similarly, the Chinese hired
on occasion
Portuguese musketeers to do some fighting for them, and they
got their Jesuit
theologian-mechanicians to make them cannon. These cannon
seem to have been
among the best the Chinese had, so good compared to the run-
of-the-foundry
product that some were still in use in the nineteenth century,
some 250 years later.
If most Chinese guns did not last that long, it was because they
were notoriously
unreliable, more dangerous to the men who fired them than to
the enemy. We even
have one report of the use of clumps of dried mud as
cannonballs. These at least
had the merit of allowing the force of the explosion to exit by
the mouth of the
tube. In general, the authorities frowned on firearms, perhaps
because they
doubted the loyalty of their subjects (Cipolla, 1966, especially
pp. 116–119).7 In
view of the inefficacy of these pieces, one wonders what they
had to fear. Presum-
ably the improvement that comes with use.
All of this may seem irrational to a means-ends oriented person,
but it was not
quite that; the ends were different. The European may have
thought that the
6 Students of the history of Chinese technology and science,
most notably Joseph Needham and his
team, have made much of Chinese priority in discovery and
invention, pushing the origins of important
techniques and devices far back, well before their appearance in
Europe. They see this quite properly
as a sign of exceptional creativity and precocity, as discussed
earlier in this paper, but they would do well
then to ask why the subsequent retreat and loss.
7 Cipolla (1966) is not a sinologist and had to rely exclusively
on European sources, including the
testimony of Christian missionaries and travelers, but his
“global vision” gives him crucial insights that
are missing in the specialist literature. Guns, Sails, and Empires
is a remarkable book.
David S. Landes 13
purpose of war was to kill the enemy and win; the Chinese,
strong in space and
numbers, thought otherwise. Here is Mu Fu-sheng (1963, pp.
76–77, a pseudonym
cited in Cipolla, 1966, p. 120) on the imperial viewpoint:
. . . military defeat was the technical reason why Western
knowledge should be
acquired, but it was also the psychological reason why it should
not be.
Instinctively the Chinese preferred admitting military defeat,
which could be
reversed, to entering a psychological crisis; people could stand
humiliation
but not self-debasement . . . . The mandarins sensed the threat
to Chinese
civilization irrespective of the economic and political issues,
and they tried to
resist this threat without regard to the economic and political
dangers. In the
past the Chinese had never had to give up their cultural pride:
the foreign
rulers always adopted the Chinese civilization. Hence there was
nothing in
their history to guide them through their modern crisis.
Along with Chinese indifference to technology went
imperviousness to European
science. The same conditions applied. The Jesuits and other
Christian clerics
brought in not only clocks but (sometimes obsolete) knowledge
and ideas. Some of
this was of interest to the court: in particular, astronomy and
techniques of celestial
observation were extremely valuable to a ruler who claimed a
monopoly of the
calendar and used his mastery of time to impose on the society
as a whole. The
Jesuits, moreover, trained gifted Chinese students who went on
to do their own
work: mathematicians who learned to use logarithms and
trigonometry and astron-
omers who prepared new star tables.
Little of this got beyond Peking, however, and the pride some
took in the new
learning was soon countered by a nativist reaction that reached
back to long-
forgotten work of earlier periods. One leader of this return to
the sources, Wen-
Ting (1635–1721), examined the texts of mathematicians who
had worked under
the Song dynasty (10th–13th centuries) and proclaimed that the
Jesuits had not
brought much in the way of innovation. Later on, his
manuscripts were published
by his grandson under the title “Pearls Recovered from the Red
River” (as discussed
in Taton, 1963–1966, volume 2, p. 592). The title was more
eloquent than intended:
by this time much of Chinese scientific “inquiry” took the form
of raking alluvial
sediment.
Meanwhile European science marched ahead, and successive
churchmen
brought to China better knowledge than their predecessors
(though still well
behind the frontier). Here, however, the churchmen were
thwarted by the con-
straints of their mission. The Christian missionaries had laid so
much stress on the
link between scientific knowledge and religious truth that any
revision of the
former implied a repudiation of the latter. When in 1710 a
Jesuit astronomer
sought to use new planetary tables based on the Copernican
system, his superior
would not permit it, for fear of “giving the impression of a
censure on what our
predecessors had so much trouble to establish and occasioning
new accusations
against [the Christian] religion” (Taton, 1963–1966, volume 2,
p. 590).
To recall these many instances of intellectual xenophobia is not
to imply that
14 Journal of Economic Perspectives
all Chinese were hostile to European ideas. We know that a few
far-sighted officials
and at least one emperor understood that the empire had much
to gain by learning
new ways.8 They were thwarted, however, not only by the
studied complacency of an
insecure superiority—also by a sense of completeness9—but by
the intrigue of a
palace milieu where innovations were judged by their
consequences for the balance
of power and influence. No proposals were made that did not
incite resistance; no
novelties offered that did not frighten vested interests. At all
levels, moreover, fear
of reprimand (or worse) outweighed the prospect of reward. A
good idea brought
credit to one’s superior; a mistake was invariably the fault of
subordinates.
One consequence was a prudent, almost instinctive, resistance
to change. This
is the heart of the matter: the response to difference and change.
The Jesuit
missionary Louis Le Comte (1655–1728) deplored this
conservatism (as quoted in
Cipolla, 1966, p. 120): “They are more fond of the most
defective piece of antiquity
than of the most perfect of the modern, differing much in that
from us [Europe-
ans], who are in love with nothing but what is new.” George
Staunton, secretary to
what is called the Macartney embassy from Great Britain to
China from 1792 to
1794, disheartened by Chinese indifference to suggestions for
improvement of
their canals, lamented (Macartney, 1804, volume 6, p. 6), “In
this country they
think that everything is excellent and that proposals for
improvement would be
superfluous if not blameworthy.” A half-century later a
Christian friar, Evariste Huc
(1844–1846, volume 6, p. 81), discouraged perhaps by the
sisyphean task of
missionizing, despairingly observed: “Any man of genius is
paralyzed immediately
by the thought that his efforts will win him punishment rather
than rewards.”
Another consequence was a plague of lies and misinformation:
officials wrote
and told their superiors what they wanted to hear; or what the
subordinate thought
the superior would want to hear.10 The smothering of incentive
and the cultivation
of mendacity are characteristic weaknesses of large
bureaucracies, whether public
or private (business corporations). These are composed of
nominal colleagues,
who are supposedly pulling together but in fact are adversarial
players. What is
more, they compete within the organization, not in a free market
of ideas, but in
a closed world of guile and maneuver. Here the advantage lies
with those in place.
Reformers and subversives beware.
The rejection of foreign technology was the more serious
because China itself
had long slipped into a regime of technological and scientific
inertia, coasting
along on the strength of previous gains and slowly losing speed
as a result of the
8 The curse of foreignness remained though. In a letter of
November 1640, the Jesuit von Bell wrote:
“The word hsi [Western] is very unpopular, and the Emperor in
his edicts never uses any word than hsin
[new]; in fact the former word in used only by those who want
to belittle us” (Taton, 1963–1966,
volume 2, p. 589, n. 1).
9 For a discussion in this spirit, see Crone (1989, pp. 172–173):
“China is a star example of a successful
civilization. . . . China reached the pinnacle of economic
development possible under pre-industrial
conditions and stopped: no forces pushing it in a different
direction are in evidence. . . .”
10 This is one of the major contributions of Peyrefitte’s (1992)
book. Because he gained access to the
Chinese archives, including papers read and annotated by the
emperor, Peyrefitte is able to show the
inner workings of bureaucratic equivocation and offer a
valuable case study.
Why Europe and the West? Why Not China? 15
inevitable frictions of vested interest and diversion of talent and
wealth into the
comfort and gratification of gentility. It has been argued that
such retirements from
the fray should not deter ambitious newcomers; on the contrary,
the prospect of
happy exits should encourage entry, and departures should make
room for others.
But in most aristocratic societies, the availability of more
esteemed careers seems to
divert talent from commerce and industry by offering short cuts
to high status. The
withdrawal of successful merchants into land and office is seen
as a logical promo-
tion, a legitimate escape. In such circumstances, the presence of
groups precluded
by birth (thus merchants in Tokugawa Japan) or belief
(Protestant dissenters in
England) from access to office and honors—the existence, in
other words, of a
reserved pool of talent—may paradoxically be a strong
contribution to otherwise
inhibited economic development.
Why Did China “Fail”?
One of the great mysteries of Chinese history is why China did
not produce
from within the kind of scientific and industrial revolutions that
gave Europe world
dominion. A thousand years ago, the Chinese were well ahead
of anyone else and
certainly of Europe. Some would argue that this superiority held
for centuries
thereafter. Why, then, did China “fail”?
Some China scholars would mitigate the pain by euphemism, as
in Fairbank
and Reischauer (1960, p. 291, cited in Oshima, 1987, p. 34):
“Chinese society,
though stable, was far from static and unchanging . . . the pace
was slower . . . the
degree of change less . . .”11 (True, but the issue remains.)
Others would dismiss
the question as unanswerable or illegitimate. Unanswerable
because it is said to be
impossible to explain a negative. (This is certainly not true in
logic; the explanation
of large-scale failure and success is inevitably complicated, but
that is what history
is all about.) Illegitimate because where is the failure? The very
use of the word
imposes non-Chinese standards and expectations on China. (But
why not? Why
should one not expect China to be interested in economic
growth and develop-
ment? To be curious about nature and want to understand it? To
want to do more
work with less labor? The earlier successes of China in these
respects make these
questions the more pertinent and acute.)
What about the relations between science and technology? Did
the one matter
to the other? After all, science was not initially a major
contributor to the European
Industrial Revolution, which was built largely on empirical
technological advances
by practitioners. What difference, then, to Chinese practitioner
technology if
science had slowed to a crawl by the seventeenth century?
The answer, I think, is that in both China and Europe, science
and technology
were (and are) two sides of the same coin, two manifestations of
a common
11 Indeed, Fairbank and Reischauer (1960) suggest that the
reason for Chinese “stability” was “the very
perfection that Chinese culture and social organization had
achieved by the thirteenth century.” The
contrast with Europe, roiling with imperfection, could not be
sharper.
16 Journal of Economic Perspectives
approach to problems and experience. The response to new
knowledge of either
kind is of a piece, and the society that closes its eyes to novelty
from one source has
already been closing them to novelty from the other.
In addition, China lacked the institutions that made for a
cumulative process
of finding and learning: the schools, the academies, the learned
societies, the
challenges and competitions. The sense of give-and-take, of
standing on the shoul-
ders of giants, of collective as well as individual achievement,
of an inherited but
ever imperfect treasure, of progress—all of these were weak or
absent in China.
And this is another paradox. On the one hand, the Chinese
formally worshiped
their intellectual ancestors; in 1734 an Imperial decree required
court physicians to
make ritual sacrifices to their departed predecessors (Taton,
1963–1966, volume 2,
p. 590). On the other, the Chinese showed a deplorable tendency
to let the findings
of each new generation slip into oblivion, to be recovered
perhaps at a later date
by antiquarian and archaeological research.12
The history of Chinese advances, then, is one of points of light,
separated in
space and time, unlinked by replication and testing, obfuscated
by metaphor and
pseudo-profundity, limited in diffusion (with no technology for
diffusion compa-
rable to European printing)—in effect, a succession of
ephemera. Much of the
technical vocabulary was invented for the occasion and fell as
swiftly into disuse; so
that later scholars spent much of their effort trying to decipher
these otherwise
familiar ideograms. Much thought remained mired in
metaphysical skepticism and
speculation. Here Confucianism, with its easy disdain for
scientific research, which
it disparaged as “interventionist” and superficial, contributed its
discouraging word.
A poem written in the early nineteenth century by the son of the
then–prime
minister, himself a high state dignitary, warned (as quoted in
Taton, 1963–1966,
volume 2, p. 593): “With the microscope you see the surface of
things. . . . But do
not suppose you are seeing the things in themselves.”13
The effect was discredit or indifference to science and
technology, the greater
for the want of mutual verification and support. This want of
continuing intellec-
tual exchange and reinforcement, this subjectivity, is what more
than anything
explains the uncertainty of scientific gains and the easy loss of
impetus. Chinese
savants had no way of knowing when they were right. It is
subsequent research,
mostly Western, that has discovered and awarded palms of
achievement to the more
inspired.
Small wonder that China reacted so unfavorably to European
imports. Euro-
pean knowledge was not only strange and implicitly belittling.
In its ebullience and
excitement, its urgency and competitiveness, its brutal
commitment to truth and
efficacy (Jesuits excepted), it went against the Chinese mindset.
12 This ongoing slippage happened in spite of considerable
effort to collect knowledge and present it in
encyclopedias. One such project, really a kind of anthology,
may well have been the biggest project of
its kind ever attempted: 800,000 pages (Spence, 1990, p. 86).
But a plethora of encyclopedias is a bad
sign: like still photographs, they are an effort to fix knowledge
at a point of time. They are useful as
reference works, especially for historians, but they can impede
free inquiry.
13 Of course, when the time came, one could find support in
Confucianism for other positions. That is
the nature of sacred writ: one can quote it to one’s purpose.
David S. Landes 17
So the years passed, and the decades, and the centuries. China
saw Europe
leave it far behind. At first China was unbelieving and
contemptuous. Later it
became increasingly anxious and frustrated. From asking and
begging, the West-
erners became insistent and impatient. The British sent two
embassies to China
seeking improved trade relations: one headed by George
Macartney in 1792 and a
second headed by William Pitt Amherst in 1816. An underlying
difficulty was that
the Chinese were happy to sell to the British, but it was very
difficult for the British
to sell to the Chinese, except for silver and opium. After a
series of diplomatic and
trade confrontations, the First Opium War started in 1839. The
British victory in
that war resulted in the Treaty of Nanjing in 1842, which
opened up Chinese ports
to British ships, reduced Chinese tariffs on British goods, and
ceded Hong Kong to
the British.
“There is Nothing We Lack”
Now England is paying homage.
My Ancestors’ merit and virtue must have reached their distant
shores.
Though their tribute is commonplace, my heart approves
sincerely.
Curios and the boasted ingenuity of their devices I prize not.
Though what they bring is meager, yet,
In my kindness to men from afar I make generous return,
Wanting to preserve my good health and power.
Poem by the Qienlong Emperor on the occasion of the
Macartney
embassy (1793)
The Empire of China is an old, crazy, first rate man-of-war,
which a
fortunate succession of able and vigilant officers has contrived
to keep
afloat these one hundred and fifty years past, and to overawe
their
neighbours by her bulk and appearance, but whenever an
insufficient
man happens to have the command upon deck, adieu to the
discipline
and safety of the ship. She may perhaps not sink outright; she
may drift
some time as a wreck, and will then be dashed to pieces on the
shore; but
she can never be rebuilt on the old bottom.
George, Lord Macartney to his journal (cited in Welsh, 1993, p.
33)
The Chinese policy of superior indifference to Western things
has been
traditionally summed up in the dismissive letter of the Qienlong
emperor (reigned
1736–1795) to George III, rejecting the British request of 1793
for trading rights
and a permanent legation in Peking: “We have never set much
store on strange and
ingenious objects, nor do we need any more of your country’s
manufactures.” So
much for scientific instruments and technological devices. That
is what I would call
potent prose. It was by no means the only such contemptuous
dismissal or trivial-
18 Journal of Economic Perspectives
ization of foreign art and artifacts during these centuries of
active contact (1550–
1900). Thus, the Qienlong Emperor’s successor, receiving and
dismissing Macart-
ney’s successor Lord Amherst in 1816, told him in effect to get
lost: “My dynasty
attaches no value to products from abroad; your nation’s
cunningly wrought and
strange wares do not appeal to me in the least” (as quoted in
Sahlins, 1988,
pp. 10–11). These explicit expressions of contempt, coming as
they did from the
emperor himself, leave little room for extenuation. The
historian, even the apol-
ogist, must deal with them—as the British had to. (They came
back in 1839 with
gunboats.)
Yet the argument has now been put forward that these back-of-
the-hand
dismissals were not a rejection of Western knowledge, but
rather messages for
internal consumption. The Manchu dynasty then ruling China
was foreign, its
legitimacy open to question. It could not afford to nourish its
enemies by admitting
to a lack of autonomy, an inferiority to other outsiders. (This
very fear of yielding—
the definition of learning as weakness!—is testimony in my
opinion to cultural
defensiveness and introversion.) In fact, this thesis continues,
the Chinese were very
much interested in Western techniques and artifacts, especially
in the military
realm. What they did not want to import was European
ideologies; and these two,
technology and ideology, were closely linked. It was the
Christian missionaries who
had done that, using, as we have seen, European knowledge and
devices to suggest
the superiority of European religion (Waley-Cohen, 1993). But
this argument is not
sustained by the facts nor is it persuasive in logic.
As to the facts: the Chinese long preceded the Europeans in the
use of
explosive powder, whether for display (fireworks) or use in
weapons. Yet a study of
their armament reveals a singular inability to enhance, by
implication an indiffer-
ence to, the destructive capacity of their bombards and cannon,
to the point where
they wreaked more fright than damage. Their very names bore
witness to their
inefficacy: thus we have the “nine-arrows, heart-penetrating,
magically poisonous
fire-thunderer,” a tube designed to blow a cluster of arrows in
the direction of the
enemy. Joseph Needham (1979) recognizes that these could not
have gone very far,
“since the gunpowder was not exerting its full propellant force.”
But he conjectures
that they might have some effect in close combat against lightly
armored or
unshielded personnel. Or the “eight-sided magical, awe-
inspiring wind-and-fire
cannon,” a vase-shaped bombard used to blow rubble and
rubbish. Too bad those
opposing these devices could not be told of their potent,
magical, awe-inspiring
names; they might have surrendered on the spot.14
Nor can one demonstrate a sustained and effective interest in
European
military technology by pointing to occasional instances of
recourse to advice and
14 The Chinese use of hyperbole in describing weaponry seems
to be a convention, and historians would
be well advised to contain their credulity. We have an account
of firearms and explosives in the later
Ming period that speaks of cannon that “when they strike a city
wall can reduce it instantly to rubble”;
and of bombards whose sighting devices are so accurate that one
“might pick off a general or remove
a prince,” as quoted in Elvin (1973, p. 94). For critical
comments on the value of this weaponry, see Sivin
(1978, p. 468). Elvin in fact is reasonably skeptical, if only
because he wants to know why the Chinese
started so fast and then slowed down.
Why Europe and the West? Why Not China? 19
technique from Jesuit missionaries. These good clerics were
ready, in the cause of
Home4 Why Europe and not China1. Why does Landes think that Chi.docx
Home4 Why Europe and not China1. Why does Landes think that Chi.docx
Home4 Why Europe and not China1. Why does Landes think that Chi.docx
Home4 Why Europe and not China1. Why does Landes think that Chi.docx
Home4 Why Europe and not China1. Why does Landes think that Chi.docx
Home4 Why Europe and not China1. Why does Landes think that Chi.docx
Home4 Why Europe and not China1. Why does Landes think that Chi.docx
Home4 Why Europe and not China1. Why does Landes think that Chi.docx
Home4 Why Europe and not China1. Why does Landes think that Chi.docx
Home4 Why Europe and not China1. Why does Landes think that Chi.docx
Home4 Why Europe and not China1. Why does Landes think that Chi.docx

More Related Content

Similar to Home4 Why Europe and not China1. Why does Landes think that Chi.docx

Worldhistory finals-trans1
Worldhistory finals-trans1Worldhistory finals-trans1
Worldhistory finals-trans1ReyesErica1
 
The Making Of A Global World
The Making Of A Global WorldThe Making Of A Global World
The Making Of A Global WorldAkanksha Akode
 
The Making Of A Global World
The Making Of A Global WorldThe Making Of A Global World
The Making Of A Global WorldAkanksha Akode
 
Industrialization Notes
Industrialization NotesIndustrialization Notes
Industrialization NotesColleen Skadl
 
Summary - Globalization - A Short History
Summary - Globalization - A Short HistorySummary - Globalization - A Short History
Summary - Globalization - A Short HistoryAlberto Rocha
 
Ch. 17 industrial revolution
Ch. 17 industrial revolutionCh. 17 industrial revolution
Ch. 17 industrial revolutionlesah2o
 
Bjmc i, dcm,unit-i, the great revolution
Bjmc i, dcm,unit-i, the great revolutionBjmc i, dcm,unit-i, the great revolution
Bjmc i, dcm,unit-i, the great revolutionRai University
 
Industrial revolution
Industrial revolutionIndustrial revolution
Industrial revolutionEdwin Juan
 
Georg franz willing - the origins of the second world war - journal of histor...
Georg franz willing - the origins of the second world war - journal of histor...Georg franz willing - the origins of the second world war - journal of histor...
Georg franz willing - the origins of the second world war - journal of histor...RareBooksnRecords
 
Making of global world
Making of global worldMaking of global world
Making of global worldankit thakur
 
AP WORLD HISTORY: Chapter 17 Revolutions of Industrialization 1750-1914
AP WORLD HISTORY: Chapter 17  Revolutions of Industrialization  1750-1914AP WORLD HISTORY: Chapter 17  Revolutions of Industrialization  1750-1914
AP WORLD HISTORY: Chapter 17 Revolutions of Industrialization 1750-1914S Sandoval
 
AP WH Chapter 26 PowerPoint
AP WH Chapter 26 PowerPointAP WH Chapter 26 PowerPoint
AP WH Chapter 26 PowerPointBruce Mulford
 
Chapter 26 ppt - Balance of Power
Chapter 26 ppt - Balance of PowerChapter 26 ppt - Balance of Power
Chapter 26 ppt - Balance of PowerAPWorldHistory
 
The decline and_fall_of_the_british_empire-robert_briffault-1938-270pgs-pol
The decline and_fall_of_the_british_empire-robert_briffault-1938-270pgs-polThe decline and_fall_of_the_british_empire-robert_briffault-1938-270pgs-pol
The decline and_fall_of_the_british_empire-robert_briffault-1938-270pgs-polRareBooksnRecords
 

Similar to Home4 Why Europe and not China1. Why does Landes think that Chi.docx (16)

Worldhistory finals-trans1
Worldhistory finals-trans1Worldhistory finals-trans1
Worldhistory finals-trans1
 
The Making Of A Global World
The Making Of A Global WorldThe Making Of A Global World
The Making Of A Global World
 
The Making Of A Global World
The Making Of A Global WorldThe Making Of A Global World
The Making Of A Global World
 
Industrialization Notes
Industrialization NotesIndustrialization Notes
Industrialization Notes
 
Part5 and6pptx
Part5 and6pptxPart5 and6pptx
Part5 and6pptx
 
Summary - Globalization - A Short History
Summary - Globalization - A Short HistorySummary - Globalization - A Short History
Summary - Globalization - A Short History
 
Ch. 17 industrial revolution
Ch. 17 industrial revolutionCh. 17 industrial revolution
Ch. 17 industrial revolution
 
Bjmc i, dcm,unit-i, the great revolution
Bjmc i, dcm,unit-i, the great revolutionBjmc i, dcm,unit-i, the great revolution
Bjmc i, dcm,unit-i, the great revolution
 
Industrial revolution
Industrial revolutionIndustrial revolution
Industrial revolution
 
Georg franz willing - the origins of the second world war - journal of histor...
Georg franz willing - the origins of the second world war - journal of histor...Georg franz willing - the origins of the second world war - journal of histor...
Georg franz willing - the origins of the second world war - journal of histor...
 
Making of global world
Making of global worldMaking of global world
Making of global world
 
AP WORLD HISTORY: Chapter 17 Revolutions of Industrialization 1750-1914
AP WORLD HISTORY: Chapter 17  Revolutions of Industrialization  1750-1914AP WORLD HISTORY: Chapter 17  Revolutions of Industrialization  1750-1914
AP WORLD HISTORY: Chapter 17 Revolutions of Industrialization 1750-1914
 
AP WH Chapter 26 PowerPoint
AP WH Chapter 26 PowerPointAP WH Chapter 26 PowerPoint
AP WH Chapter 26 PowerPoint
 
Chapter 26 ppt - Balance of Power
Chapter 26 ppt - Balance of PowerChapter 26 ppt - Balance of Power
Chapter 26 ppt - Balance of Power
 
The decline and_fall_of_the_british_empire-robert_briffault-1938-270pgs-pol
The decline and_fall_of_the_british_empire-robert_briffault-1938-270pgs-polThe decline and_fall_of_the_british_empire-robert_briffault-1938-270pgs-pol
The decline and_fall_of_the_british_empire-robert_briffault-1938-270pgs-pol
 
Ap ch 22
Ap ch 22Ap ch 22
Ap ch 22
 

More from pooleavelina

httpswww.azed.govoelaselpsUse this to see the English Lang.docx
httpswww.azed.govoelaselpsUse this to see the English Lang.docxhttpswww.azed.govoelaselpsUse this to see the English Lang.docx
httpswww.azed.govoelaselpsUse this to see the English Lang.docxpooleavelina
 
httpscdnapisec.kaltura.comindex.phpextwidgetpreviewpartner_.docx
httpscdnapisec.kaltura.comindex.phpextwidgetpreviewpartner_.docxhttpscdnapisec.kaltura.comindex.phpextwidgetpreviewpartner_.docx
httpscdnapisec.kaltura.comindex.phpextwidgetpreviewpartner_.docxpooleavelina
 
httpsifes.orgsitesdefaultfilesbrijuni18countryreport_fi.docx
httpsifes.orgsitesdefaultfilesbrijuni18countryreport_fi.docxhttpsifes.orgsitesdefaultfilesbrijuni18countryreport_fi.docx
httpsifes.orgsitesdefaultfilesbrijuni18countryreport_fi.docxpooleavelina
 
httpfmx.sagepub.comField Methods DOI 10.117715258.docx
httpfmx.sagepub.comField Methods DOI 10.117715258.docxhttpfmx.sagepub.comField Methods DOI 10.117715258.docx
httpfmx.sagepub.comField Methods DOI 10.117715258.docxpooleavelina
 
httpsiexaminer.orgfake-news-personal-responsibility-must-trump.docx
httpsiexaminer.orgfake-news-personal-responsibility-must-trump.docxhttpsiexaminer.orgfake-news-personal-responsibility-must-trump.docx
httpsiexaminer.orgfake-news-personal-responsibility-must-trump.docxpooleavelina
 
http1500cms.comBECAUSE THIS FORM IS USED BY VARIOUS .docx
http1500cms.comBECAUSE THIS FORM IS USED BY VARIOUS .docxhttp1500cms.comBECAUSE THIS FORM IS USED BY VARIOUS .docx
http1500cms.comBECAUSE THIS FORM IS USED BY VARIOUS .docxpooleavelina
 
httpswww.medicalnewstoday.comarticles323444.phphttpsasco.docx
httpswww.medicalnewstoday.comarticles323444.phphttpsasco.docxhttpswww.medicalnewstoday.comarticles323444.phphttpsasco.docx
httpswww.medicalnewstoday.comarticles323444.phphttpsasco.docxpooleavelina
 
httpstheater.nytimes.com mem theater treview.htmlres=9902e6.docx
httpstheater.nytimes.com mem theater treview.htmlres=9902e6.docxhttpstheater.nytimes.com mem theater treview.htmlres=9902e6.docx
httpstheater.nytimes.com mem theater treview.htmlres=9902e6.docxpooleavelina
 
httpsfitsmallbusiness.comemployee-compensation-planThe pu.docx
httpsfitsmallbusiness.comemployee-compensation-planThe pu.docxhttpsfitsmallbusiness.comemployee-compensation-planThe pu.docx
httpsfitsmallbusiness.comemployee-compensation-planThe pu.docxpooleavelina
 
httpsdoi.org10.11770002764219842624American Behaviora.docx
httpsdoi.org10.11770002764219842624American Behaviora.docxhttpsdoi.org10.11770002764219842624American Behaviora.docx
httpsdoi.org10.11770002764219842624American Behaviora.docxpooleavelina
 
httpsdoi.org10.11770896920516649418Critical Sociology.docx
httpsdoi.org10.11770896920516649418Critical Sociology.docxhttpsdoi.org10.11770896920516649418Critical Sociology.docx
httpsdoi.org10.11770896920516649418Critical Sociology.docxpooleavelina
 
httpsdoi.org10.11770894318420903495Nursing Science Qu.docx
httpsdoi.org10.11770894318420903495Nursing Science Qu.docxhttpsdoi.org10.11770894318420903495Nursing Science Qu.docx
httpsdoi.org10.11770894318420903495Nursing Science Qu.docxpooleavelina
 
httpswww.youtube.comwatchtime_continue=8&v=rFV0aes0vYAN.docx
httpswww.youtube.comwatchtime_continue=8&v=rFV0aes0vYAN.docxhttpswww.youtube.comwatchtime_continue=8&v=rFV0aes0vYAN.docx
httpswww.youtube.comwatchtime_continue=8&v=rFV0aes0vYAN.docxpooleavelina
 
httphps.orgdocumentspregnancy_fact_sheet.pdfhttpswww.docx
httphps.orgdocumentspregnancy_fact_sheet.pdfhttpswww.docxhttphps.orgdocumentspregnancy_fact_sheet.pdfhttpswww.docx
httphps.orgdocumentspregnancy_fact_sheet.pdfhttpswww.docxpooleavelina
 
httpswww.worldbank.orgencountryvietnamoverview---------.docx
httpswww.worldbank.orgencountryvietnamoverview---------.docxhttpswww.worldbank.orgencountryvietnamoverview---------.docx
httpswww.worldbank.orgencountryvietnamoverview---------.docxpooleavelina
 
HTML WEB Page solutionAbout.htmlQuantum PhysicsHomeServicesAbou.docx
HTML WEB Page solutionAbout.htmlQuantum PhysicsHomeServicesAbou.docxHTML WEB Page solutionAbout.htmlQuantum PhysicsHomeServicesAbou.docx
HTML WEB Page solutionAbout.htmlQuantum PhysicsHomeServicesAbou.docxpooleavelina
 
httpswww.huffpost.comentryonline-dating-vs-offline_b_4037867.docx
httpswww.huffpost.comentryonline-dating-vs-offline_b_4037867.docxhttpswww.huffpost.comentryonline-dating-vs-offline_b_4037867.docx
httpswww.huffpost.comentryonline-dating-vs-offline_b_4037867.docxpooleavelina
 
httpswww.vitalsource.comproductscomparative-criminal-justice-.docx
httpswww.vitalsource.comproductscomparative-criminal-justice-.docxhttpswww.vitalsource.comproductscomparative-criminal-justice-.docx
httpswww.vitalsource.comproductscomparative-criminal-justice-.docxpooleavelina
 
httpswww.nationaleatingdisorders.orglearnby-eating-disordera.docx
httpswww.nationaleatingdisorders.orglearnby-eating-disordera.docxhttpswww.nationaleatingdisorders.orglearnby-eating-disordera.docx
httpswww.nationaleatingdisorders.orglearnby-eating-disordera.docxpooleavelina
 
httpswww.youtube.comwatchtime_continue=59&v=Bh_oEYX1zNM&featu.docx
httpswww.youtube.comwatchtime_continue=59&v=Bh_oEYX1zNM&featu.docxhttpswww.youtube.comwatchtime_continue=59&v=Bh_oEYX1zNM&featu.docx
httpswww.youtube.comwatchtime_continue=59&v=Bh_oEYX1zNM&featu.docxpooleavelina
 

More from pooleavelina (20)

httpswww.azed.govoelaselpsUse this to see the English Lang.docx
httpswww.azed.govoelaselpsUse this to see the English Lang.docxhttpswww.azed.govoelaselpsUse this to see the English Lang.docx
httpswww.azed.govoelaselpsUse this to see the English Lang.docx
 
httpscdnapisec.kaltura.comindex.phpextwidgetpreviewpartner_.docx
httpscdnapisec.kaltura.comindex.phpextwidgetpreviewpartner_.docxhttpscdnapisec.kaltura.comindex.phpextwidgetpreviewpartner_.docx
httpscdnapisec.kaltura.comindex.phpextwidgetpreviewpartner_.docx
 
httpsifes.orgsitesdefaultfilesbrijuni18countryreport_fi.docx
httpsifes.orgsitesdefaultfilesbrijuni18countryreport_fi.docxhttpsifes.orgsitesdefaultfilesbrijuni18countryreport_fi.docx
httpsifes.orgsitesdefaultfilesbrijuni18countryreport_fi.docx
 
httpfmx.sagepub.comField Methods DOI 10.117715258.docx
httpfmx.sagepub.comField Methods DOI 10.117715258.docxhttpfmx.sagepub.comField Methods DOI 10.117715258.docx
httpfmx.sagepub.comField Methods DOI 10.117715258.docx
 
httpsiexaminer.orgfake-news-personal-responsibility-must-trump.docx
httpsiexaminer.orgfake-news-personal-responsibility-must-trump.docxhttpsiexaminer.orgfake-news-personal-responsibility-must-trump.docx
httpsiexaminer.orgfake-news-personal-responsibility-must-trump.docx
 
http1500cms.comBECAUSE THIS FORM IS USED BY VARIOUS .docx
http1500cms.comBECAUSE THIS FORM IS USED BY VARIOUS .docxhttp1500cms.comBECAUSE THIS FORM IS USED BY VARIOUS .docx
http1500cms.comBECAUSE THIS FORM IS USED BY VARIOUS .docx
 
httpswww.medicalnewstoday.comarticles323444.phphttpsasco.docx
httpswww.medicalnewstoday.comarticles323444.phphttpsasco.docxhttpswww.medicalnewstoday.comarticles323444.phphttpsasco.docx
httpswww.medicalnewstoday.comarticles323444.phphttpsasco.docx
 
httpstheater.nytimes.com mem theater treview.htmlres=9902e6.docx
httpstheater.nytimes.com mem theater treview.htmlres=9902e6.docxhttpstheater.nytimes.com mem theater treview.htmlres=9902e6.docx
httpstheater.nytimes.com mem theater treview.htmlres=9902e6.docx
 
httpsfitsmallbusiness.comemployee-compensation-planThe pu.docx
httpsfitsmallbusiness.comemployee-compensation-planThe pu.docxhttpsfitsmallbusiness.comemployee-compensation-planThe pu.docx
httpsfitsmallbusiness.comemployee-compensation-planThe pu.docx
 
httpsdoi.org10.11770002764219842624American Behaviora.docx
httpsdoi.org10.11770002764219842624American Behaviora.docxhttpsdoi.org10.11770002764219842624American Behaviora.docx
httpsdoi.org10.11770002764219842624American Behaviora.docx
 
httpsdoi.org10.11770896920516649418Critical Sociology.docx
httpsdoi.org10.11770896920516649418Critical Sociology.docxhttpsdoi.org10.11770896920516649418Critical Sociology.docx
httpsdoi.org10.11770896920516649418Critical Sociology.docx
 
httpsdoi.org10.11770894318420903495Nursing Science Qu.docx
httpsdoi.org10.11770894318420903495Nursing Science Qu.docxhttpsdoi.org10.11770894318420903495Nursing Science Qu.docx
httpsdoi.org10.11770894318420903495Nursing Science Qu.docx
 
httpswww.youtube.comwatchtime_continue=8&v=rFV0aes0vYAN.docx
httpswww.youtube.comwatchtime_continue=8&v=rFV0aes0vYAN.docxhttpswww.youtube.comwatchtime_continue=8&v=rFV0aes0vYAN.docx
httpswww.youtube.comwatchtime_continue=8&v=rFV0aes0vYAN.docx
 
httphps.orgdocumentspregnancy_fact_sheet.pdfhttpswww.docx
httphps.orgdocumentspregnancy_fact_sheet.pdfhttpswww.docxhttphps.orgdocumentspregnancy_fact_sheet.pdfhttpswww.docx
httphps.orgdocumentspregnancy_fact_sheet.pdfhttpswww.docx
 
httpswww.worldbank.orgencountryvietnamoverview---------.docx
httpswww.worldbank.orgencountryvietnamoverview---------.docxhttpswww.worldbank.orgencountryvietnamoverview---------.docx
httpswww.worldbank.orgencountryvietnamoverview---------.docx
 
HTML WEB Page solutionAbout.htmlQuantum PhysicsHomeServicesAbou.docx
HTML WEB Page solutionAbout.htmlQuantum PhysicsHomeServicesAbou.docxHTML WEB Page solutionAbout.htmlQuantum PhysicsHomeServicesAbou.docx
HTML WEB Page solutionAbout.htmlQuantum PhysicsHomeServicesAbou.docx
 
httpswww.huffpost.comentryonline-dating-vs-offline_b_4037867.docx
httpswww.huffpost.comentryonline-dating-vs-offline_b_4037867.docxhttpswww.huffpost.comentryonline-dating-vs-offline_b_4037867.docx
httpswww.huffpost.comentryonline-dating-vs-offline_b_4037867.docx
 
httpswww.vitalsource.comproductscomparative-criminal-justice-.docx
httpswww.vitalsource.comproductscomparative-criminal-justice-.docxhttpswww.vitalsource.comproductscomparative-criminal-justice-.docx
httpswww.vitalsource.comproductscomparative-criminal-justice-.docx
 
httpswww.nationaleatingdisorders.orglearnby-eating-disordera.docx
httpswww.nationaleatingdisorders.orglearnby-eating-disordera.docxhttpswww.nationaleatingdisorders.orglearnby-eating-disordera.docx
httpswww.nationaleatingdisorders.orglearnby-eating-disordera.docx
 
httpswww.youtube.comwatchtime_continue=59&v=Bh_oEYX1zNM&featu.docx
httpswww.youtube.comwatchtime_continue=59&v=Bh_oEYX1zNM&featu.docxhttpswww.youtube.comwatchtime_continue=59&v=Bh_oEYX1zNM&featu.docx
httpswww.youtube.comwatchtime_continue=59&v=Bh_oEYX1zNM&featu.docx
 

Recently uploaded

Introduction to ArtificiaI Intelligence in Higher Education
Introduction to ArtificiaI Intelligence in Higher EducationIntroduction to ArtificiaI Intelligence in Higher Education
Introduction to ArtificiaI Intelligence in Higher Educationpboyjonauth
 
Full Stack Web Development Course for Beginners
Full Stack Web Development Course  for BeginnersFull Stack Web Development Course  for Beginners
Full Stack Web Development Course for BeginnersSabitha Banu
 
Alper Gobel In Media Res Media Component
Alper Gobel In Media Res Media ComponentAlper Gobel In Media Res Media Component
Alper Gobel In Media Res Media ComponentInMediaRes1
 
Organic Name Reactions for the students and aspirants of Chemistry12th.pptx
Organic Name Reactions  for the students and aspirants of Chemistry12th.pptxOrganic Name Reactions  for the students and aspirants of Chemistry12th.pptx
Organic Name Reactions for the students and aspirants of Chemistry12th.pptxVS Mahajan Coaching Centre
 
KSHARA STURA .pptx---KSHARA KARMA THERAPY (CAUSTIC THERAPY)————IMP.OF KSHARA ...
KSHARA STURA .pptx---KSHARA KARMA THERAPY (CAUSTIC THERAPY)————IMP.OF KSHARA ...KSHARA STURA .pptx---KSHARA KARMA THERAPY (CAUSTIC THERAPY)————IMP.OF KSHARA ...
KSHARA STURA .pptx---KSHARA KARMA THERAPY (CAUSTIC THERAPY)————IMP.OF KSHARA ...M56BOOKSTORE PRODUCT/SERVICE
 
CELL CYCLE Division Science 8 quarter IV.pptx
CELL CYCLE Division Science 8 quarter IV.pptxCELL CYCLE Division Science 8 quarter IV.pptx
CELL CYCLE Division Science 8 quarter IV.pptxJiesonDelaCerna
 
MARGINALIZATION (Different learners in Marginalized Group
MARGINALIZATION (Different learners in Marginalized GroupMARGINALIZATION (Different learners in Marginalized Group
MARGINALIZATION (Different learners in Marginalized GroupJonathanParaisoCruz
 
Incoming and Outgoing Shipments in 1 STEP Using Odoo 17
Incoming and Outgoing Shipments in 1 STEP Using Odoo 17Incoming and Outgoing Shipments in 1 STEP Using Odoo 17
Incoming and Outgoing Shipments in 1 STEP Using Odoo 17Celine George
 
Earth Day Presentation wow hello nice great
Earth Day Presentation wow hello nice greatEarth Day Presentation wow hello nice great
Earth Day Presentation wow hello nice greatYousafMalik24
 
internship ppt on smartinternz platform as salesforce developer
internship ppt on smartinternz platform as salesforce developerinternship ppt on smartinternz platform as salesforce developer
internship ppt on smartinternz platform as salesforce developerunnathinaik
 
Introduction to AI in Higher Education_draft.pptx
Introduction to AI in Higher Education_draft.pptxIntroduction to AI in Higher Education_draft.pptx
Introduction to AI in Higher Education_draft.pptxpboyjonauth
 
“Oh GOSH! Reflecting on Hackteria's Collaborative Practices in a Global Do-It...
“Oh GOSH! Reflecting on Hackteria's Collaborative Practices in a Global Do-It...“Oh GOSH! Reflecting on Hackteria's Collaborative Practices in a Global Do-It...
“Oh GOSH! Reflecting on Hackteria's Collaborative Practices in a Global Do-It...Marc Dusseiller Dusjagr
 
Capitol Tech U Doctoral Presentation - April 2024.pptx
Capitol Tech U Doctoral Presentation - April 2024.pptxCapitol Tech U Doctoral Presentation - April 2024.pptx
Capitol Tech U Doctoral Presentation - April 2024.pptxCapitolTechU
 
Software Engineering Methodologies (overview)
Software Engineering Methodologies (overview)Software Engineering Methodologies (overview)
Software Engineering Methodologies (overview)eniolaolutunde
 
भारत-रोम व्यापार.pptx, Indo-Roman Trade,
भारत-रोम व्यापार.pptx, Indo-Roman Trade,भारत-रोम व्यापार.pptx, Indo-Roman Trade,
भारत-रोम व्यापार.pptx, Indo-Roman Trade,Virag Sontakke
 
Hierarchy of management that covers different levels of management
Hierarchy of management that covers different levels of managementHierarchy of management that covers different levels of management
Hierarchy of management that covers different levels of managementmkooblal
 
Painted Grey Ware.pptx, PGW Culture of India
Painted Grey Ware.pptx, PGW Culture of IndiaPainted Grey Ware.pptx, PGW Culture of India
Painted Grey Ware.pptx, PGW Culture of IndiaVirag Sontakke
 

Recently uploaded (20)

Introduction to ArtificiaI Intelligence in Higher Education
Introduction to ArtificiaI Intelligence in Higher EducationIntroduction to ArtificiaI Intelligence in Higher Education
Introduction to ArtificiaI Intelligence in Higher Education
 
Full Stack Web Development Course for Beginners
Full Stack Web Development Course  for BeginnersFull Stack Web Development Course  for Beginners
Full Stack Web Development Course for Beginners
 
Alper Gobel In Media Res Media Component
Alper Gobel In Media Res Media ComponentAlper Gobel In Media Res Media Component
Alper Gobel In Media Res Media Component
 
Organic Name Reactions for the students and aspirants of Chemistry12th.pptx
Organic Name Reactions  for the students and aspirants of Chemistry12th.pptxOrganic Name Reactions  for the students and aspirants of Chemistry12th.pptx
Organic Name Reactions for the students and aspirants of Chemistry12th.pptx
 
OS-operating systems- ch04 (Threads) ...
OS-operating systems- ch04 (Threads) ...OS-operating systems- ch04 (Threads) ...
OS-operating systems- ch04 (Threads) ...
 
KSHARA STURA .pptx---KSHARA KARMA THERAPY (CAUSTIC THERAPY)————IMP.OF KSHARA ...
KSHARA STURA .pptx---KSHARA KARMA THERAPY (CAUSTIC THERAPY)————IMP.OF KSHARA ...KSHARA STURA .pptx---KSHARA KARMA THERAPY (CAUSTIC THERAPY)————IMP.OF KSHARA ...
KSHARA STURA .pptx---KSHARA KARMA THERAPY (CAUSTIC THERAPY)————IMP.OF KSHARA ...
 
CELL CYCLE Division Science 8 quarter IV.pptx
CELL CYCLE Division Science 8 quarter IV.pptxCELL CYCLE Division Science 8 quarter IV.pptx
CELL CYCLE Division Science 8 quarter IV.pptx
 
MARGINALIZATION (Different learners in Marginalized Group
MARGINALIZATION (Different learners in Marginalized GroupMARGINALIZATION (Different learners in Marginalized Group
MARGINALIZATION (Different learners in Marginalized Group
 
Incoming and Outgoing Shipments in 1 STEP Using Odoo 17
Incoming and Outgoing Shipments in 1 STEP Using Odoo 17Incoming and Outgoing Shipments in 1 STEP Using Odoo 17
Incoming and Outgoing Shipments in 1 STEP Using Odoo 17
 
Earth Day Presentation wow hello nice great
Earth Day Presentation wow hello nice greatEarth Day Presentation wow hello nice great
Earth Day Presentation wow hello nice great
 
internship ppt on smartinternz platform as salesforce developer
internship ppt on smartinternz platform as salesforce developerinternship ppt on smartinternz platform as salesforce developer
internship ppt on smartinternz platform as salesforce developer
 
Introduction to AI in Higher Education_draft.pptx
Introduction to AI in Higher Education_draft.pptxIntroduction to AI in Higher Education_draft.pptx
Introduction to AI in Higher Education_draft.pptx
 
“Oh GOSH! Reflecting on Hackteria's Collaborative Practices in a Global Do-It...
“Oh GOSH! Reflecting on Hackteria's Collaborative Practices in a Global Do-It...“Oh GOSH! Reflecting on Hackteria's Collaborative Practices in a Global Do-It...
“Oh GOSH! Reflecting on Hackteria's Collaborative Practices in a Global Do-It...
 
Capitol Tech U Doctoral Presentation - April 2024.pptx
Capitol Tech U Doctoral Presentation - April 2024.pptxCapitol Tech U Doctoral Presentation - April 2024.pptx
Capitol Tech U Doctoral Presentation - April 2024.pptx
 
Software Engineering Methodologies (overview)
Software Engineering Methodologies (overview)Software Engineering Methodologies (overview)
Software Engineering Methodologies (overview)
 
भारत-रोम व्यापार.pptx, Indo-Roman Trade,
भारत-रोम व्यापार.pptx, Indo-Roman Trade,भारत-रोम व्यापार.pptx, Indo-Roman Trade,
भारत-रोम व्यापार.pptx, Indo-Roman Trade,
 
Model Call Girl in Bikash Puri Delhi reach out to us at 🔝9953056974🔝
Model Call Girl in Bikash Puri  Delhi reach out to us at 🔝9953056974🔝Model Call Girl in Bikash Puri  Delhi reach out to us at 🔝9953056974🔝
Model Call Girl in Bikash Puri Delhi reach out to us at 🔝9953056974🔝
 
Hierarchy of management that covers different levels of management
Hierarchy of management that covers different levels of managementHierarchy of management that covers different levels of management
Hierarchy of management that covers different levels of management
 
Painted Grey Ware.pptx, PGW Culture of India
Painted Grey Ware.pptx, PGW Culture of IndiaPainted Grey Ware.pptx, PGW Culture of India
Painted Grey Ware.pptx, PGW Culture of India
 
Model Call Girl in Tilak Nagar Delhi reach out to us at 🔝9953056974🔝
Model Call Girl in Tilak Nagar Delhi reach out to us at 🔝9953056974🔝Model Call Girl in Tilak Nagar Delhi reach out to us at 🔝9953056974🔝
Model Call Girl in Tilak Nagar Delhi reach out to us at 🔝9953056974🔝
 

Home4 Why Europe and not China1. Why does Landes think that Chi.docx

  • 1. Home4 Why Europe and not China? 1. Why does Landes think that China would not have developed an industrial revolution on its own? (Landes 2006 “Why Europe and the West? Why not China?” is posted on file) 2. Why does he think that China failed to learn new technologies from Europeans in the period after 1500? 3. In Landes’ view, what did Europe have that China lacked? That is, what did Europe have that permitted it to have an industrial revolution? 4. What does Pomeranz say about the factors that Landes identifies as the crucial features of European society that permitted it to have an industrial revolution? Why does he say that these features did not matter? 5. What does Pomeranz think are the crucial factors that enabled Europe to have an industrial revolution? Note: You can learn about Pomeranz’s ideas from Marks, pp 104-118.(Already posted it on file) required that all goods be transported in their ships, and forced European New World colonists to trade only with the mother country, even if smuggling made such a policy somewhat porous. Mercantilist ideas also led to policies that states should use their own raw materials to manufacture within their own borders anything that was imported, an action we saw the English take in the early 1700s to keep Indian cotton textiles out. Although mercantilist policies did indeed lead to the
  • 2. establishment of industries in European states, industrialization itself was not the object: keeping gold and silver from flowing out of the state and enriching others was. European states were obsessed with their silver stocks: ‘‘the more silver, the stronger the state’’ was how a German once put it.40 In these inter-European wars, the fates and fortunes of various states rose and fell. As we have already seen, by the end of the sixteenth century, Spain’s power had begun to wane, and Portugal proved to be too small to mount much of a challenge to the French (or Spanish) in Europe, or to the Dutch in Asian waters. The Dutch, being among the first Europeans to apply vast amounts of capital to their trading enterprises in both Asia and the Americas, saw their fortunes peak in the seventeenth century, just as the French and the British were gaining power. Ultimately, though, the Dutch did not have the manpower to build a standing army sufficiently large to counter the French, and they ultimately allied with the British to offset French power on the continent. By the eighteenth century, Britain and France had emerged from the seventeenth-century crisis as the two most powerful and competitive European states. (See map 3.1.)
  • 3. The Seven Years’ War, 1756–1763 As the strongest and most successful European states, England and France competed not just in Europe but in the Americas and Asia as well. In the ‘‘long’’ eighteenth century from 1689 to 1815, Britain and France fought five wars, only one of which Britain did not initiate. Their engagement (with others) in the War of Spanish Succession was ended by the 1713 Treaty of Utrecht, which established the principle of the ‘‘balance of power’’ in Europe, that is, that no country should be allowed to dominate the others. However, periodic wars between the British and French continued. 104 But the most significant was the Seven Years’ War of 1756– 1763, or what Americans call the French and Indian War and interpret in terms of its impact on the American War of Independence of 1776–1783 against Britain. To be sure, the spark that led to war between Britain and France came in the American colonies, and it was in fact the twenty- two-year-old George Washington who lit it.41 But it became a global engagement— perhaps the first real world war—with British and French troops
  • 4. fighting in the backwoods of the American colonies, in Canada, in Africa, in India, and in Europe. The outcome was disastrous for the French: they lost their colonial claims in both North America (the British got Canada) and in India, leading to greater British power and position in both parts of the world.42 By 1775, therefore, the processes of state building in Europe had led to the creation of a system defined by war, which favored a particular kind of state exemplified by the ones built in Britain and France. Balance of power among sovereign states, not a unified empire, had become the established principle, and Britain had emerged as the strongest European state. But that does not mean that it was the strongest or richest state in the world— far from it. To be sure, Mughal power in India was declining in the early 1700s, and as we will see in the next chapter, the British were able to begin building a colonial empire there. But the British were still too weak to be able to contest China’s definition of the rules of trade in Asia. When they tried, most famously in 1793 under Lord Macartney’s mission, the Chinese emperor sent them home with a stinging rebuke, and the British could do nothing about it. However, the British Isles were
  • 5. fortunate enough to be the location for the start of the Industrial Revolution, which 105 was gaining steam even as Lord Macartney was sailing back to London. And when the British learned to apply the tools of the Industrial Revolution to war, the global balance of power between Britain and China tipped. That is the story of the next chapter. 106 CHAPTER FOUR The Industrial Revolution and Its Consequences, 1750–1850 In 1750, nearly all of the world’s 750 million people, regardless of where they were or what political or economic system they had, lived and died within the biological old regime. The necessities of life—food, clothing, shelter, and fuel for heating and cooking—mostly came from the land, from what could be captured from annual energy flows from the sun to Earth.1 Industries too, such as textiles, leather, and construction, depended
  • 6. on products from agriculture or the forest. Even iron and steel making in the biological old regime, for instance, relied upon charcoal made from wood. The biological old regime thus set limits not just on the size of the human population but on the productivity of the economy as well. These limits would begin to be lifted over the century from 1750 to 1850, when some people increasingly used coal to produce heat and then captured that heat to fuel repetitive motion with steam-powered machines, doing work that previously had been done with muscle. The use of coal- fired steam to power machines was a major breakthrough, launching human society out of the biological old regime and into a new one no longer limited by annual solar energy flows. Coal is stored solar energy, laid down hundreds of millions of years ago. Its use in steam engines freed human society from the limits imposed by the biological old regime, enabling the productive powers and numbers of humans to grow exponentially. The replacement—with steam generated by burning coal— 107 of wind, water, and animals for powering industrial machines
  • 7. constitutes the beginning of the Industrial Revolution2 and ranks with the much earlier agricultural revolution in importance for the course of history. The use of fossil fuels—first coal and then petroleum—not only transformed economies around the world but also added greenhouse gases to Earth’s atmosphere. How and why this major transformation happened, and what consequences it had, thus are vitally important matters in world history and will be the focus of this and the following two chapters. To understand the Industrial Revolution, we will use once again the tool of conjuncture, that is, the coming together at a particular point in time of otherwise separate historical developments and processes. In the case of the Industrial Revolution, the conjuncture involves the playing out around the world of growth potential in the biological old regime, the extension of European state conflicts around the globe, the peculiar nature of New World colonies, and the chance location of, and challenges for operating, coal mines in England. In particular, I will consider the ways cotton textiles and the British need for coal contributed to the Industrial Revolution. Cotton Textiles
  • 8. The Industrial Revolution is commonly thought to have begun in eighteenth-century England with the mechanization of the process for spinning and weaving cotton thread and cloth. The spinning jenny, the water frame, and the ‘‘mule’’ all have been taken as evidence of English inventiveness and hence contribute to a Eurocentric story line of the rise of the West. The problem is, while it is true that England was the first place to revolutionize cotton manufacture by using steam-powered machinery, how and why it happened can only be understood in a global context.3 In the late seventeenth century, the English developed a strong desire for the Indian cotton textiles commonly known as calicoes. As one man observed: ‘‘On a sudden we saw all our women, rich and poor, cloath’d in Callico, printed and painted; the gayer the better.’’ Another complained: ‘‘It crept into our houses, our closets and bedchambers; curtains, cushions, chairs, and at last beds themselves were nothing but Callicoes or Indian stuffs. In short, almost everything that used to be made of wool or silk, relating either to dress of the women or the furniture of our houses, was supplied by the Indian trade.’’4 These observations by contemporaries around 1700 raise some interesting questions: Why were the
  • 9. English importing so much Indian cotton? How did it get there? How did they then create and industrialize a cotton textile industry? 108 The reason the English imported so much Indian cotton around 1700 is it was of high quality and lower price than England’s domestically produced textiles (in particular linen and wool). It felt good next to the skin, it was lightweight for summer wear, it could accept bright dyes for color, and most of all, it was less expensive than anything the English could manufacture themselves. Indeed, India around 1700 was the largest exporter of cotton textiles in the world and supplied textiles not just to meet English demand, but throughout the world as well. Southeast Asia, East and West Africa, the Middle East, and Europe were major export markets, in addition to the large domestic Indian market. No wonder that the demand for Indian cotton in the eighteenth century was ‘‘greater than all the weavers in the country can manufacture,’’ and that India accounted for fully one-quarter of the world manufacturing output in 1750.5
  • 10. Like so many things desired by Europeans and supplied by Asians—at first luxury items for the elite such as silk or porcelain, but increasingly products like tea from China for a mass market6 —cotton textiles were produced well and cheaply in India. The British textile manufacturers focused on the ‘‘cheap’’ part and complained that with relatively higher wages, British manufacturers could not compete. India had a competitive advantage in the eighteenth century, being able to undersell in the world market virtually any other producer of textiles. Some thought the reason for cheap Indian textiles was a low living standard, or a large population earning depressed wages, but all of those have been shown to not be true: Indian textile workers in the eighteenth century had just as high a standard of living as British workers.7 So, if it was not a low standard of living that gave India its competitive advantage, what did? In a word: agriculture. Indian agriculture was so productive that the amount of food produced, and hence its cost, was significantly lower than in Europe. In the preindustrial age, when working families spent 60–80 percent of their earnings on food, the cost of food was the primary determinant of their real wages (i.e., how much a pound, a dollar, a real, or
  • 11. a pagoda could buy). In India (and China and Japan as well), the amount of grain harvested from a given amount of seed was in the ratio of 20:1 (e.g., twenty bushels of rice harvested for every one planted), whereas in England it was at best 8:1. Asian agriculture thus was more than twice as efficient as British (and by extension European) agriculture, and food—the major component in the cost of living—cost less in Asia. Thus although nominal wages may have been lower in India, the purchasing power—the real wage—was higher in India. In the biological old regime, productive agriculture was Asia’s competitive advantage, even in industry. The causal chain went like this: 109 high per-acre yields V low-priced food V relatively low wages V comparative advantage. In England, the causal chain was like this: low per-acre yields V high-priced food V relatively high wages V comparative disadvantage. The question then becomes, how did the British begin to reverse this comparative disadvantage? In part, as we saw in the previous chapter, they did it by raising tariffs
  • 12. on imports to Britain of Indian textiles, and the outright banning of the importation of some kinds of Indian cotton goods—that is, mercantilist protectionism. Had the British not done that in the early eighteenth century, there is little reason to believe they would have made much progress in competing against Indian producers and establishing much of a cotton textile industry in the first place.8 But also, the British had colonies in the Americas and acquired their ‘‘jewel’’ in India. Both became intimately connected with the story of the rise of a cotton textile industry in Britain. India Indeed, where England had very little by way of overseas empire in 1650, it soon began putting one together, preying on Portuguese and Spanish possessions in the East and West Indies (i.e., India and the Caribbean), competing with the Dutch in both regions of the world, and battling France in the eighteenth century. Curiously, though, the agents for this extension of European interstate conflict around the world were not at first the governments of European states but private trading companies, the first being the Dutch Vereenigde Oost-Indische Compagnie (VOC, East India Company), the English East India Company (EIC), and the
  • 13. Compagnie française des Indes occidentales (French West India Company). Although each was formed at different times and had slightly different organization, all were private companies chartered by their governments and given monopoly rights to trade with Asia, all in keeping with mercantilist ideas. They also differed from mere trading expeditions in that they were formed with a permanent capital and stock that could be traded —to that extent, the East India companies are the forerunners of the modern corporation, and their success at organizing trade and raising profits meant that the corporation would play an increasingly important role in European industrialization. But in the seventeenth and eighteenth centuries, their purpose was to reap profits from trade with Asia. The Dutch VOC, though, seeing itself as an extension of Protestant Dutch interests and hence deeply hostile to the Catholic powers of Spain and Portugal, saw trade and war as intimately connected. In a terse 1614 letter to his directors, the Dutch VOC governor-general observed: ‘‘You 110
  • 14. gentlemen ought to know from experience that trade in Asia should be conducted and maintained under the protection and with the aid of your own weapons, and . . . [s]o trade cannot be maintained without war, nor war without trade.’’9 The Dutch then effectively pursued this strategy throughout the seventeenth century, taking Melaka from the Portuguese, seizing Java and making it into a sugar-producing colony, and trying to establish a colony on the Chinese island of Taiwan. The English EIC, by contrast, was more interested in trade and the profits of trade than in war, at least at first. In the century after its founding in 1600, the directors insisted that ‘‘our business is trade, not war.’’10 To avoid conflicts, the English EIC concentrated trade in India, where Indian states were weak and European competitors few, especially in Bengal and Madras. But by the late seventeenth century, that began to change as the French established forts nearby. And when the British and French warred in Europe, their forces (however small) clashed in India, with the French usually getting the upper hand because they began enhancing their war- making capability by enlisting Indians as regulars, known as Sepoys, into their army. In the 1750s the British EIC followed suit, and by
  • 15. the eve of the Seven Years’ War, each had nearly ten thousand men in arms—mostly Indians—on the Indian coast. In the meantime, the political and military power of the great Mughal empire had seriously declined. At its height it was capable of mobilizing perhaps a million troops; after the death of its last great leader, Aurangzeb, in 1707, the empire declined as regional political and military leaders asserted their independence from the Mughals. One of those leaders, the nawab of Bengal, took control of the British trading port at Kolkata (Calcutta) and demanded increased payments from the EIC for the privilege of trading there. The British resisted, sent a force of some two thousand men under the leadership of Robert Clive and, together with other Indian forces opposed to Bengal, defeated the nawab’s French-assisted forces at the Battle of Plassey in 1757. They captured and executed the nawab, got a more pliable replacement, and by 1765 received the right to collect tax revenue—a huge sum—from Bengal. In the meantime, of course, the Seven Years’ War had begun, and British and French forces had at it up and down the Indian coast, with the British winning a decisive victory over the
  • 16. French at Pondicherry in 1760. This was the start of the British empire in India, and over the next fifty years the extent of British control widened, with the entire subcontinent becoming a formal colony in 1857. (See map 3.1.) The Seven Years’ War—or more precisely, the British victory in the Americas and in India—is important to the story of how Britain became a 111 cotton-textile-producing, rather than -importing, country. Recall that the British government had banned the importation of Indian textiles in 1707 for the purpose of allowing its domestic cotton industry to get going, which it did, in the area around the town of Lancaster. But because of technical difficulties in copying Indian dyeing techniques and because of its higher wages/higher prices, Lancashire (the region around Lancaster) produced mostly for the British home market, still being bested in the world market by Indian textiles traded by the EIC. For the British cotton textile industry to grow, it therefore needed export markets. And there was a growing market in the New World because of its peculiar
  • 17. institutions of slavery, plantations, and mercantilist trade restrictions. The New World as a Peculiar Periphery European New World agriculture from the beginning was export oriented. Throughout the Caribbean and South America, mostly all sugar, tobacco, and cotton was produced on plantations that used African slaves because of labor shortages caused by the Great Dying and the unwillingness of Europeans to migrate to the New World. Unlike peasants in India and China or serfs in eastern Europe, African slaves in America did not grow much of their own food. Food, especially fish and grain, had to be imported, mostly from the North American colonies. Slaves also had to be clothed, creating a demand for cheap cotton textiles. Additional quantities of Indian textiles were traded in West Africa for slaves who were then sold in the Caribbean. New World products—sugar, tobacco, raw cotton—were taken back to England.11 At each point in the triangular Atlantic trade, the English made profits and by colonial legislation tried to ensure that the New World would remain producers of raw materials only and consumers of the industrial products of Britain. Smuggling or trading with the enemy, whether Dutch
  • 18. or French, was pervasive, but by the early eighteenth century, ‘‘colonial trade conformed in almost every particular to the navigation system . . . [and] smuggled goods accounted for a tiny fraction of all quantities handled.’’ Of course, the colonists in both the Caribbean and North America were Englishmen, and they too looked for ways to profit from a system that denied other nationals, especially the Dutch or the French, from getting a piece of Britain’s colonial trade.12 This triangular trade and in particular the linkage between the slave trade and textiles fueled the growth of British shipping and established Lancashire as a center of cotton textile manufacture. Raw cotton was imported mostly from the Levant in the Ottoman empire and the British colonies in the Caribbean, and by the 1780s it was spun into thread in 112 newfangled ‘‘factories’’ using water power and employing hundreds of workers in one place. As the Lancashire manufacturers became more proficient and the prices of their textiles declined, they even exported them to Africa, especially whenever Indian textiles were expensive.
  • 19. The real boom to British cotton textile production came after American independence in 1793 when Eli Whitney’s invention of the cotton gin made it possible to use short-staple and much cheaper American cotton. With another series of innovations derived from the application of steam power, as we will see shortly, mechanizing both spinning and weaving over the years from 1815 to 1840, the productivity of the Lancashire textile factories surged again, resulting in ever lower prices and the ability to outcompete Indian textiles in the world market. Indian textile producers had created a worldwide market for cotton textiles—the British then captured it.13 When that happened, the British became advocates of ‘‘free trade’’ and abandoned both mercantilist theory and practice and tariffs on imports. Indeed, ‘‘free trade’’ became the ideological mantra of imperial Britain at the height of its global power in the nineteenth century. Mercantilism, at least as it applied in the Atlantic world, had been dead since the victory of the Americans in their War of Independence from Britain. To the British, their former American subjects and colonists became ‘‘foreigners, subject to all the provisions of the Navigation Laws,’’14 which restricted the
  • 20. importation of raw cotton, potentially strangling the British textile industry and giving rise to calls for ‘‘free trade.’’ Free trade with the new United States after 1783 showed the fallacy of the argument that British manufactures could grow only with a monopoly on colonial markets, and the American South with its cotton plantations worked by African slaves and their descendants became the major supplier of raw cotton to the mills of Lancashire. Although this story of the rise to global competitiveness of the British cotton textile industry sounds Eurocentric, it really is not, for British success was contingent upon a number of worldwide developments that were not of their own making. In the first place, the British were at a competitive disadvantage to Indian producers and would have remained so except for several coincidences. The Glorious Revolution of 1688–1689 brought to power a government willing to use state power to protect its domestic manufacturers; and the New World developed as a peculiar periphery that, by the accident of the Great Dying and colonial legislation, provided a market for British manufactured goods. In the second place, the British were fortunate to develop a usable coal-fueled steam engine, which
  • 21. further revolutionized cotton textile production, making it even more 113 productive and its products so cheap that the British could undersell Indian textiles not just in Africa but, interestingly, in India as well. For that part of the story, we now look at the innovations in coal and steam engines. New Sources of Energy and Power Until about 1830, the story of cotton textiles for the most part remains one that unfolds within the biological old regime; that is, everything about it depended on the annual flows of solar energy and their capture by humans.15 To be sure, the early British ‘‘factories’’ had begun to use water power, but there was a limit to how much that could increase cotton textile production. Indeed, there is every reason to think that cotton textile production would have reached serious limits within the biological old regime, leading not to an industrial revolution but to an economic dead end, had it not been for coal, the steam engine, and iron and steel production, which truly launched the Industrial Revolution and allowed Britain to break out of the constraints imposed by the biological
  • 22. old regime. To see how and why, we need to take a closer look at what was happening to the most advanced biological old regime economies, starting with China and then looking at England. What we will see is that all old regime economies were beginning to push up against serious ecological constraints that would have stopped all of them from developing an industrial revolution. Except for a few chance occurrences and a vast global conjuncture, we all now might still be living in the biological old regime. China Two favored explanations of the Industrial Revolution in Europe have focused on population dynamics and the growth of free markets. By various techniques and practices, mostly late marriage, European families were able to keep their sizes smaller than ‘‘naturally’’ possible. Smaller family size meant a smaller population overall, leaving greater surpluses in the hands of families to invest in improving agricultural and industrial productivity. Fewer people working harder to make their investable surpluses grow—an ‘‘industrious revolution,’’ it is said—grew inexorably into the Industrial Revolution.16
  • 23. The market-driven story line of industrialization suggests that the establishment and growth of markets for commodities, land, labor, and capital in Europe enabled European producers to be much more efficient and hence to accumulate sufficient capital to invest in improving agricultural and industrial productivity. Also necessary for the success of 114 markets was a state that protected (or at least respected) private property rights. This combination likewise, according to the Eurocentric version of the origins of the modern world, grew more or less naturally into the Industrial Revolution. Of course, the population- and market-driven story lines of industrialization are not incompatible, and many historians have melded them together in explaining why Europeans were uniquely capable of launching an industrial revolution. As proof, they often point to China as a counterexample. China, it is alleged, had ‘‘a preindustrial demographic regime,’’ in which nothing was done to keep birthrates down. Hence, population surged, eating up any surplus above subsistence and rendering
  • 24. the investments necessary for an industrial revolution impossible.17 Similarly, it is alleged, China was ‘‘despotic’’: it had a state that meddled in private affairs, property rights were not respected, and markets could not operate efficiently. Hence, it is concluded, there was no possibility for an industrial revolution. There is only one thing wrong with these assumptions about what ‘‘went wrong’’ in China: they are wrong. As I will show below, Chinese families in fact had numerous ways—albeit different from Europeans—of limiting their size and hence keeping the overall Chinese population above subsistence levels. Also, Chinese markets of all kinds not merely existed, but arguably functioned better and more efficiently than those in Europe. If both of those are true for China, then their value as ‘‘explanations’’ for why the Industrial Revolution occurred in Europe is questionable. To see why, we must take a closer look at China. As mentioned earlier in this chapter, agriculture in China (as well as in India and numerous other parts of Asia) was highly productive, harvesting twenty bushels of rice for every one sown. Rice has the unique capability of gaining nutrients not directly from the soil, but from the water (and so it
  • 25. is grown in ‘‘paddies’’), eliminating the need for the land to lie fallow, as was the custom in Europe, to regain its fertility. Additionally, Chinese farmers had learned how to prepare the soil, to irrigate, to fertilize, and to control insect pests in order to maximize the harvest yield. Moreover, farmers in the southern half of China could get two or sometimes three harvests per year from the same plot of land, drawing the amazement of early-eighteenth-century European travelers to China. ‘‘By what art can the earth produce subsistence for such numbers [of people]?’’ asked the Frenchman Pierre Poivre in the 1720s. Do the Chinese possess any secret arts of multiplying grain and provisions necessary for the nourishment of mankind? To solve my 115 doubts I traversed the fields, I introduced myself among the laborers, who are in general easy, polite, and knowledgeable of the world. I examine, and pursue them through all their operations, and observe that their secret consists simply in manuring the fields judiciously, ploughing them to a considerable depth, sowing them in the proper season, turning to
  • 26. advantage every inch of ground which can produce the most considerable crop, and preferring to every other species of culture that of grain, as by far the most important.18 Such an impressively productive agriculture certainly allowed the Chinese population to grow, from 140 million in 1650, to 225 million in 1750, and then to 380–400 million by 1850. Numbers like these also convinced European observers, in particular Adam Smith and Thomas Malthus, whose ideas about markets and population have so shaped Eurocentric views of the modern world, that the Chinese just could not control their population growth. Malthus believed that populations like the Chinese who could not control their growth would overshoot the capability of the land to support their numbers until ‘‘negative’’ population checks, such as famines or wars, reduced the population size. Malthus also believed that Europeans avoided those fates by having ‘‘preventative’’ checks on population growth. Where Malthus certainly was right about Europeans, he was wrong about the Chinese. The fact is, they could—and did—control their family size, although in ways quite different from the Europeans. Although
  • 27. almost all Chinese women married and married early, Chinese families developed many methods for controlling the number of children. Abstention from having sexual relations, especially early in the marriage, was a preferred mechanism and was enforced by married couples living with their parents. Infanticide, especially of daughters, was another means to limit family size, leading as well to a gender-unbalanced population of more men than women, and hence of forced celibacy for many poorer men. As James Lee and Wang Feng summarize the Chinese demographic system: In contrast to the European system, in which marriage was the only volitional check on population growth, the Chinese demographic system had multiple conscious checks, and was therefore far more complex and calculating than Malthus or his successors thought. As a result . . . population never pushed the economy to subsistence levels.19 Nevertheless, because of the productivity of agriculture and the ability of the Chinese economy to produce more than enough food for its 116
  • 28. population, the population did in fact grow, and as mentioned above, it grew rapidly from 1750 to 1850.20 In the densely populated core areas of the Pearl River delta in south China, along the southeast coast, and in the Yangzi River delta, populations did reach the size where people started migrating out into less-populated areas. Sometimes these regions had exceptionally fertile soil that could be brought into production by clearing the land, as in Hunan up the Yangzi River from Shanghai, or in the West River valley in Guangxi province, or sometimes the land that was brought into production was more marginal and less fertile, as in the Jiangxi highlands on the southern bank of the Yangzi River.21 Wherever new land was being brought into agricultural production, especially by 1800 when it was land that was not as fertile or productive as land in the densely populated core regions, that was an indication that the limits of growth within the biological old regime were being reached. That does not mean that a Malthusian disaster was imminent—the Chinese were in fact very much in control of their reproductive capabilities— but that good agricultural land was becoming in short supply. The reason for this is that the four necessities of life—food, clothing, shelter, and fuel—all came
  • 29. from the land and hence were in competition. Clearing land for food decreased the amount of wood available for fuel, either to cook and heat homes or to make charcoal for industrial purposes. Switching land from cotton to rice production also put pressure on the supply of the raw material for clothing, while doing the opposite would decrease the amount of food available. There just was not much room for maneuvering when the limits of the biological old regime were being reached, as they were in China in the late eighteenth century,22 and, as we will see, in Britain too. It was not just that meeting the needs for sustained population growth meant increasing pressure on the land and decreases in other things at the expense of food, but that to keep food production increasing while at the same time keeping supplies available for clothing, shelter, and fuel meant that greater and greater amounts of labor and capital had to be expended in agriculture just to keep pace. For instance, clearing land was expensive and so too was building irrigation works or terracing fields from hills, all of which improved the output of Chinese agriculture in the eighteenth and nineteenth centuries. Allocating more labor also could increase output, and Chinese farming families did that too: planting rice in nurseries
  • 30. and then transplanting it to the fields or picking insects off rice plants by hand, for instance, also increased agricultural yields and sustained a growing population. So too did capturing and recycling nitrogen from human and animal waste; Asian farmers were biological-old-regime champions in maintaining or even increasing the fertility of their fields. 117 Markets Another way the Chinese economy improved both overall production levels and productivity was by the use of markets, especially for agricultural commodities. It used to be thought that markets were first and most highly developed in Europe (reading backward from the Industrial Revolution to find reasons why it happened there first). But in the past thirty years, historians of China have shown how fully developed and efficient markets were in eighteenth- and nineteenth-century China.23 Peasant farmers in the Pearl River and Yangzi River deltas, for instance, came to specialize in sericulture (that is, the whole process of producing silk), raising silkworms and growing the mulberry trees with which to feed the silkworms, boiling the cocoons to obtain the silk threads,
  • 31. then spinning, weaving, and dyeing the silk. Other areas might specialize in cotton, sugarcane, or other nonfood agricultural crops. Such specialization meant that those peasant producers had to obtain their food from other sources, usually places upriver that came to specialize in rice and could easily export it on boats to the more densely populated core regions. Massive investments in canals by both private parties and the state vastly extended and improved the Chinese inland water transportation system, linking China from Tianjin in the north to Guangzhou in the south by water. Efficient water transportation facilitated the movement of grain throughout the Chinese empire and the growth of markets and provided the material foundations for maintaining some of the world’s largest cities. Initially, the Chinese state intervened in the food markets quite regularly to ensure that peasant producers and urban consumers alike would be ensured adequate food supplies,24 but by the mid- eighteenth century the Chinese state was increasingly willing to allow markets and merchants to handle the movement of grain across huge distances—up to a thousand miles—from where it was produced to where it was consumed.
  • 32. Measures of the efficiency of these markets show that they were more efficient than contemporary markets in France, England, or the United States.25 Additionally, Chinese markets for land, labor, and capital all functioned well and in some ways more efficiently than comparable markets in European countries.26 In short, eighteenth-century China looked as ‘‘developed’’ as any other developed part of the world, whether measured by levels of agricultural productivity, sophistication of manufactures and markets, or levels of consumption. Chinese families regulated their size and were responsive to changing economic opportunities, limiting their size when those opportunities diminished in order to maintain consumption above 118 subsistence levels; specialization of function gave rise to markets and a highly commercialized economy; and an extensive water-based transportation system allowed the efficient movement of goods and people throughout the empire. Yet China’s highly developed market economy did not lead to an industrial breakthrough. Instead, by the nineteenth century,
  • 33. there were plenty of indicators that China was pushing up against ecological constraints imposed by the biological old regime. In several areas, fuel became in short supply in the early 1800s, with peasant families turning to rice straw and chaff for heating and cooking rather than wood. Moreover, some market exchanges between densely populated core regions and developing peripheral areas also served to slow Chinese economic growth. One of the advantages of markets and a good transportation network is that they allow some areas to specialize in what their natural resources make most profitable and to exchange that produce with others, enabling both to be more productive and allowing everyone’s income to rise. At least that is the theory, and to a point that is the way markets functioned in China. However, the exchanges began to break down regarding the trade of raw cotton from cotton-producing regions in return for manufactured goods, cotton textiles in particular, from the highly developed core regions in the lower Yangzi and Pearl River deltas. Throughout China, rural families were free to decide what and how much to grow and how to allocate family labor on the farm. To this extent,
  • 34. they differed markedly from African slaves in the New World or serfs in eastern Europe, both of whom had their freedom curtailed and production decisions made by their owners or overseers. Thus those Chinese peasants who migrated to more peripheral areas, like their counterparts in the more developed cores, were free to make their own decisions. Increasingly, what they decided was that it was in their interest to spin and weave their own cotton textiles for their own use and for local exchange, rather than concentrating on rice or raw cotton and importing the finished goods. In effect, large parts of rural China underwent a process of ‘‘import substitution,’’ producing their own textiles. Not only did they reduce the amount of raw cotton sold to the textile-producing centers, but they also increased the area given over to cotton and hence decreased the amount of rice they were willing to export as well.27 The freedom of Chinese peasant families thus may have spurred what might be called ‘‘self-sufficient proto-industrialization’’ in peripheral areas, but that acted as a constraint on the growth of an industrial cotton textile industry in China’s core regions. Contributing to the willingness of Chinese peasant families in peripheral areas to spin and weave their own
  • 35. 119 textiles may have been the long-standing norm that ‘‘men plow, women weave.’’ It was not just that ‘‘women weave,’’ but that they weave in the household. Chinese families thus placed a high value on mothers and daughters staying at home to do the weaving, rather than leaving home to work in a factory, as English and Japanese girls did.28 Paradoxically, the freedom of peasant farmers throughout China’s core and peripheral regions, when compared with the limited freedoms of slaves and serfs in the European system, constrained China’s ability to continue developing a textile industry in its most highly developed core regions. In summary, China had a highly developed market economy within the constraints of the biological old regime. Nonetheless, that regime placed ecological limits upon growth, and the freedom of Chinese peasants coupled with practices governing the sexual division of labor, all combined, meant that China was bumping up against the limits of growth by the mid-1800s. Food, clothing, shelter, and fuel competed for land, and to get more from the land, the Chinese lavished increasing
  • 36. amounts of labor on agriculture. The dynamics of specialization, increased market exchanges, and improved transportation in the context of the biological old regime and the particularities of China’s situation were pushing it toward an increasingly labor-intensive agriculture and the depletion of land resources, rather than toward an industrial revolution. As fuel supplies from wood declined, the Chinese turned again, where they could, to coal. Coal was available and used in North China, parts of Central China, and in South China too, where it was used in the iron foundries of Foshan town near the major city of Guangzhou. Nevertheless, this availability of coal in China did not lead to an industrial revolution there, or elsewhere in Eurasia for that matter. Particular circumstances came together in one place, England, that did create the conditions for the leap into the fossil-fueled industrial age that became the modern world. Exhausting the Earth29 Indeed, over the period from 1400 to 1800, the dynamics of the biological old regime, especially in those areas most densely populated and even with the institutions of private property, markets, and effective states, were leading not to a breakthrough to a ‘‘modern world’’ of fossil- fuel energy
  • 37. and electrified industry and homes, but to an increasingly intensive using up of resources. In part, this outcome was driven by a significant worldwide population increase, rising from 380 to 950 million people, most of that coming in the century following the depths of the seventeenth- century global crisis, which drove the removal of forests for more farmland (and food for people).30 Interestingly, climatologists now think 120 Why Europe and the West? Why Not China? David S. Landes T he world history of technology is the story of a long, protracted inversion.As late as the end of the first millennium of our era, the civilizations ofAsia were well ahead of Europe in wealth and knowledge. The Europe of what we call the Middle Ages (say, tenth century) had regressed from the power and pomp of Greece and Rome, had lost much of the science it had once possessed, had seen its economy retreat into generalized autarky. It traded little with other societies, for it had little surplus to sell, and insofar as it wanted goods from outside, it paid for them largely with human beings. Nothing testifies
  • 38. better to deep poverty than the export of slaves or the persistent exodus of job-hungry migrants. Five hundred years later, the tables had turned. I like to summarize the change in one tell-tale event: the Portuguese penetration into the Indian Ocean led by Vasco da Gama in 1498. This was an extraordinary achievement. Some scholars will tell you that it was some kind of accident; that it could just as easily have been Muslim sailors, or Indian, or Chinese to make the connection from the other direction. Did not the Chinese send a series of large fleets sailing west as far as the east African coast in the early fifteenth century—bigger, better and earlier than anything the Portuguese had to show? Don’t you believe it. These affirmations of Asian priority are especially prom- inent and urgent nowadays because a new inversion is bringing Asia to the fore. A “multicultural” world history finds it hard to live with a eurocentric story of achievement and transformation. So a new would-be (politically correct) orthodoxy would have us believe that a sequence of contingent events (gains by Portugal and then others in the Indian Ocean, followed by conquests by Spain and then others in the New World) gave Europe what began as a small edge and was then worked up into centuries of dominion and exploitation. A gloss on this myth contends that
  • 39. y David S. Landes is Emeritus Professor of Economics, Harvard University, Cambridge, Massachusetts. Journal of Economic Perspectives—Volume 20, Number 2— Spring 2006—Pages 3–22 a number of non-European societies were themselves on the edge of a technolog- ical and scientific breakthrough; that in effect, European tyranny (to paraphrase Thomas Gray’s “Elegy Written in a Country Churchyard”), “froze the genial current of the [Asian] soul.” A variant on this history-as-accident (or luck) is the pendulum approach associated with Jack Goody’s (1996) book, East in the West. Everything starts on an even keel thanks to the allegedly common heritage of the Bronze Age; but then different parts move ahead, only to be caught up and passed by others, which then lose ground to their predecessors. So Europe was just especially lucky, taking the lead at the crucial turn to the Industrial Revolution. But Asia’s turn will now come; indeed is already coming. As Goody (pp. 231–232) writes: “[I ]t is a pendular movement that continues today, with the East now beginning to dominate the West in matters of the economy.” As for efforts to understand this European success—
  • 40. especially explanations based on allegedly deep characteristics that were present in Europe but wanting in China—such efforts are irrelevant, writes Goody (p. 238): . . . since all these features must have been present [in China] at the earlier period. Those discussions can be seen for what they are, as representing the understandable but distorting tendency of Europeans to inflate their overall contribution to world society and even to ‘Western civilisation’, a tendency reinforced by their undoubted achievements over the past few centuries. Such inflation of oneself inevitably involves the deflation of others; self-congratulation is a zero-sum game. But of course, Westerners were not alone in noticing some European deep characteristics. Thus Abu Talib, an Indian Muslim visitor to Britain late eighteenth century, commenting on British precocity in mechanization: “The British,” he wrote (cited in Khan, 1998, p. 303), “were endowed with a natural passion for technical innovation. They possessed inventive skills and preferred to perform even minor routine jobs with the aid of mechanical instruments rather than manually. They had such great passion for the use of technical instruments that they would not perform certain tasks unless the necessary instruments were at their disposal.” The French, he went on, were not like that.1
  • 41. I shall return later to this revisionist debate. Here, suffice to say: 1) The Portuguese success was the result of decades of rational exploration and extension of navigational possibilities in an ocean (the south Atlantic) that was hostile to traditional techniques of navigation, which essentially involved following the coast- line. This technological enhancement rested in turn on a systematic utilization of astronomical observations and calculations, taken from the Muslims and transmit- ted largely by Jewish intermediaries, which allowed the Portuguese to follow winds and currents across the south Atlantic, and then use a knowledge of latitude to swing back around the tip of Africa and into the Indian Ocean. 2) The Chinese 1 Khan (1998, p. 328, n. 122) notes further that the Arabic lacked the vocabulary needed to speak of factory manufacture or machinery. For the latter, Abu Talib used “wheels and tools.” 4 Journal of Economic Perspectives abandonment of westward exploration was partly the result of contingent political events; but at bottom it reflected the values and structures of Chinese society and civilization. 3) European exploitation of the breakthrough rested on a disparity of power technology (better powder and better guns) as well as on
  • 42. navigational superiority. The extension of European power into other parts of the world was the expression of these and other disparities. Why other regions did not keep up with Europe is an important historical question, for one learns almost as much from failure as from success. It is not possible in brief compass, of course, to pose this question for every non-European society or civilization; but three do deserve serious reflection: Islam, China, and India. I shall focus in this essay on China. The First Chance: Science without Development The one civilization that was in a position to match and even anticipate the European achievement was China. China had two chances: first, to generate a continuing, self-sustaining process of scientific and technological advance on the basis of its indigenous traditions and achievements; and second, to learn from European science and technology once the foreign “barbarians” entered the Chi- nese domain in the sixteenth century. China failed both times. The first failure has elicited much scholarly inquiry and analysis. And yet it remains an abiding mystery. The China specialists tell us, for example, that in a number of areas of industrial technique, China long anticipated Europe: in textiles,
  • 43. where the Chinese had a power-driven spinning machine in the thirteenth century, some 500 years before the England of the Industrial Revolution knew water frames and mules; or in iron manufacture, where the Chinese early learned to use coal and probably coke (as against charcoal) in blast furnaces for smelting iron and were turning out perhaps as many as 125,000 tons of pig iron by the later eleventh century—a figure not achieved by Britain until 700 years later (Elvin, 1973, p. 85).2 In general, one can establish a long list of instances of Chinese priority: the wheelbarrow, the stirrup, the rigid horse collar (to prevent choking), the compass, paper, printing, gunpowder, porcelain. (But not the horse-shoe, which implies that the Chinese did not make use of the horse for transport.) The mystery lies in the failure of China to realize the potential of some of the most important of these inventions. One generally assumes that knowledge and know-how are cumulative and that a superior technique, once known, will domi- 2 Elvin (1973) gives the figure as “between 35,000 to 40,000 tons and 125,000 tons,” but says he prefers the higher estimate. He relies here on Yoshida Mitsukuni, a Japanese specialist writing in 1967. Work by Hartwell (1966, p. 34), also advances the higher figure. In Hall (1985, p. 46), this becomes “at least 125,000 tons.” In this regard, Elvin (p. 285) quotes a description by Yen Ju-yu of iron works on the
  • 44. Hupei/Shensi/Szechwan borders with blast furnaces 18 feet high, using charcoal and hand-operated bellows (more than ten persons relaying one another) and working continuously. The iron was apparently used for castings, and there is no indication of further refining as either wrought iron or steel. David S. Landes 5 nate older methods and remain in use. But Chinese industrial history offers a number of examples of technological regression and oblivion. The machine to spin hemp was never adapted to the manufacture of cotton; cotton spinning was never mechanized; and coal/coke smelting was allowed to fall into disuse, along with the iron industry. Why, asks Elvin (1973, pp. 297–298)? It would seem that none of the conventional explanations tells us in convinc- ing fashion why technical progress was absent in the Chinese economy during a period that was, on the whole, one of prosperity and expansion. Almost every element usually regarded by historians as a major contributory cause to the Indus- trial Revolution in north-western Europe was also present in China. There had even been a revolution in the relations between social classes, at least in the countryside; but this had had no important effect on the techniques of production. Only
  • 45. Galilean-Newtonian science was missing; but in the short run this was not impor- tant. Had the Chinese possessed, or developed, the seventeenth- century European mania for tinkering and improving, they could easily have made an efficient spinning machine out of the primitive model described by Wang Chen. A steam engine would have been more difficult; but it should not have posed insuperable difficulties to a people who had been building double-acting piston flame-throwers in the Sung dynasty. The crucial point is that nobody tried. In most fields, agricul- ture being the chief exception, Chinese technology stopped progressing well before the point at which a lack of scientific knowledge had become a serious obstacle. Why indeed? Sinologists have put forward several partial explanations. Those that I find most persuasive are the following. First, China lacked a free market and institutionalized property rights. The Chinese state was always stepping in to interfere with private enterprise—to take over certain activities, to prohibit and inhibit others, to manipulate prices, to exact bribes. At various times the government was motivated by a desire to reserve labor to agriculture or to control important resources (salt and iron, for example); by an appetite for revenue (the story of the goose that laid the golden eggs is a leitmotif
  • 46. of Chinese history); by fear and disapproval of self-enrichment, except by officials, giving rise in turn to abundant corruption and rent-seeking; and by a distaste for maritime trade, which the Heavenly Kingdom saw as a diversion from imperial concerns, as a divisive force and source of income inequality in the ecumenical empire, and worse yet, as an invitation to exit. This state intervention and inter- ference encountered evasion and resistance; indeed, the very needs of state com- pelled a certain tolerance for disobedience. Still, the goal, the aim, the ideal was the ineffable stillness of immobility. When in 1368 the new Chinese emperor inaugu- rated a native (Ming) dynasty to replace the defeated Mongol invaders, he as- cended the throne in Nanjing as the Hongwu (“Vast Martial”) emperor. Let not the name deceive the reader: Hongwu’s goal was anything but war. He wanted rather to immobilize the realm. People were to stay put and move only with the permission of the state—at home and abroad. People who went outside China without per- mission were liable to execution on their return. The Ming code of core laws also sought to block social mobility, with severe penalties for those jumping professional 6 Journal of Economic Perspectives and occupational barriers. In this regard, Timothy Brook (1998,
  • 47. p. vii) cites in epigraph one of the Hongwu emperor’s favorite moral dicta: Let the state be small and the people few; So that the people . . . fearing death, will be reluctant to move great distances And, even if they have boats and carts, will not use them. So that the people . . . will find their food sweet and their clothes beautiful, Will be content with where they live and happy in their customs. Though adjoining states be within sight of one another and cocks crow- ing and dogs barking in one be heard in the next, Yet the people of one state will grow old and die without having had any dealings with those of another. These matters reached a wretched climax under the Ming dynasty (1368–1644), when the state attempted to prohibit all trade overseas.3 Such interdictions led of course to evasion and smuggling, with concomitant corruption (protection money), searches for contraband, confiscations and punishment. All of this neces- sarily acted to strangle initiative, to increase risk and the cost of transactions, and to chase talent from commerce and industry.
  • 48. A second reason why China did not realize the economic potential of its scientific expertise involved the larger values of the society. The great Hungarian- German-French sinologist, Etienne Balazs (1968 [1988]; see also Balazs, 1964), saw China’s abortive technology as part of a larger pattern of totalitarian control. He recognizes the absence of freedom, along with the weight of custom and consensus and what passed for higher wisdom. His analysis (pp. 22–23) is worth repeating: . . . if one understands by totalitarianism the complete hold of the State and its executive organs and functionaries over all the activities of social life, without exception, Chinese society was highly totalitarian. . . . No private initiative, no expression of public life that can escape official control. There is to begin with a whole array of state monopolies, which comprise the great consumption staples: salt, iron, tea, alcohol, foreign trade. There is a monop- oly of education, jealously guarded. There is practically a monopoly of letters (I was about to say, of the press): anything written unofficially, that escapes the censorship, has little hope of reaching the public. But the reach of the 3 The imperial authorities vacillated in their attitude to foreign trade, now favoring it, now clamping down; and these tergiversations were in themselves a deterrent to stable enterprise and capital accu-
  • 49. mulation. In addition, even when the state relented, it did so in circumstances that pushed the traders into illicit operations. Thus, the early Mongol (Yuan) dynasty (1280–1368) allowed freedom of enter- prise, but then succumbed to the temptation of instituting a licensing system. This enabled officials to play the role of capitalist, financing venturers and dividing profits 70-30: 70 for the official, 30 for the working trader. That was greedy, compared to the typical European 50-50 split. The traders presumably sought to conceal gains, but in the long run, trade had to suffer. Why Europe and the West? Why Not China? 7 Moloch-State, the omnipotence of the bureaucracy, goes much farther. There are clothing regulations, a regulation of public and private construction (dimensions of houses); the colors one wears, the music one hears, the festivals—all are regulated. There are rules for birth and rules for death; the providential State watches minutely over every step of its subjects, from cradle to grave. It is a regime of paper work and harassment, endless paper work and endless harassment. The ingenuity and inventiveness of the Chinese, which have given so much to mankind—silk, tea, porcelain, paper, printing, and more—would no doubt have enriched China further and probably brought it to the threshold
  • 50. of modern industry, had it not been for this stifling state control. It is the State that kills technological progress in China. Not only in the sense that it nips in the bud anything that goes against or seems to go against its interests, but also by the customs implanted inexorably by the raison d’Etat. The atmosphere of routine, of traditionalism, and of immobility, which makes any innovation suspect, any initiative that is not commanded and sanctioned in advance, is unfavorable to the spirit of free inquiry. In short, to go back to Elvin (1973), the reason the Chinese did not develop based on their scientific knowledge is that no one was trying. Why try? Especially since the Chinese were not without their own quiet resources to thwart bureaucratic interferences and frustrations—reliance on personal and familial collaboration, for example, in place of arbitrary or institutional practice in business. In such matters, personal trust could yield more dependable performance than legal rules. In all this, the contrast with Europe was marked. Where fragmentation and national rivalries compelled European rulers to pay heed to their subjects, to recognize their rights and cultivate the sources of wealth, the rulers of China had a free hand. Again Elvin (1973, pp. 224–225) captures some of this:
  • 51. . . . it was the great size of the Chinese Empire which made the adoption of the policies of the Ming emperors possible. In a Chinese subcontinent made up of smaller independent states, like those of the Five Dynasties [907-960 C.E.] or the Ten Kingdoms, no government could have afforded to close itself off. International economic interdependence (as that between regions would have become) would have removed this option; and the need for diplomatic and military alliances, and revenue from foreign trade, would have made isolationism undesirable. With smaller states, there might also have been, as there was in north-western Europe in early modern times, a closer conscious identification of the governed with their countries and rulers. Prior to mod- ern communications, the immensity of the empire precluded nationalism. Whatever the mix of factors, the result seems to have been a curious pattern of isolated initiatives and sisyphean discontinuities—up, up, up and then down again—almost as though the society were constrained by a homeostatic braking mechanism or held down by a silk ceiling. The result, if not the aim, was a kind of 8 Journal of Economic Perspectives
  • 52. change-in-immobility; or maybe immobility-in-change. Innovation was allowed to go (was able to go) so far and no farther.4 The Europeans knew much less of these interferences. Instead, they entered during these centuries into an exciting world of innovation and emulation that challenged and tempted vested interests and kept the forces of conservatism scrambling. Changes were cumulative, news of novelty spread fast and a new sense of progress and achievement replaced an older, effete reverence for authority. This intoxicating sense of freedom touched (infected) all domains. These were years of heresies in the church, of popular initiatives that, we can see now, anticipated the rupture of the Reformation; of new forms of expression and collective action that challenged the older organization of society and posed a threat to other polities; of new ways of doing and making things that made newness a virtue and a source of delight. Important in all this was the role of the Christian church in Europe as custodian of knowledge and school for technicians. One might have expected otherwise: that organized spirituality, with its emphasis on prayer and contempla- tion, would have had little interest in technology; and that with its view of labor as penalty for original sin, it would have had no concern to save labor. And yet
  • 53. everything seems to have worked in the opposite direction: The desire to free clerics from time-consuming earthly tasks led to the introduction and diffusion of power machinery and, beginning with the Cistercians in the twelfth century, to the hiring of lay brothers (conversi) to do the dirty work, which led in turn to an awareness of and attention to time and productivity. All of this gave rise on monastic estates to remarkable assemblages of powered machinery—complex se- quences designed to make the most of the water power available and distribute it through a series of industrial operations. A description of the abbey of Clairvaux in the mid-twelfth century (cited in White, 1978, p. 245–246) exults in this versatility: “coquendis, cribrandis, vertendis, terendis, rigandis, lavandis, molendis, molliendis, suum sine contradictione praestans obsequium.” The author, clearly proud of these achieve- ments, further tells his readers that he will take the liberty of joking (the medieval clerical equivalent of, “if you’ll pardon the expression”): the fulling hammers, he says, seem to have dispensed the fullers of the penalty for their sins; and he thanks God that such devices can mitigate the oppressive labor of men and spare the backs of their horses. Why this peculiarly European joy in discovery? This pleasure in the new and better? This cultivation of invention—or what some have called “the invention of
  • 54. invention”? Different scholars have suggested a variety of reasons, typically related to religious values. One possible reason grows from the Judaeo- Christian respect for manual labor, summed up in a number of biblical injunctions. One example will suffice: when God warns Noah of the coming flood and tells him he will be saved, it is not God who saves him. “Build thee an ark of gopher wood,” says the Lord, and 4 For example, Max Weber (1922 [1951], as cited in Hall, 1985, p. 41) argued that the administrative bureaucracy was undermanned, so that government came to know and respond to changes only after they had gotten under way. Hence a pattern of “intermittent and jerky” homeostatic interventions. David S. Landes 9 Noah builds an ark to divine specifications. A second and related reason is the Judaeo-Christian subordination of nature to man. This belief is a sharp departure from widespread animistic beliefs and practices that saw something of the divine in every tree and stream (hence the naiads and dryads). Ecologists today might say these animistic beliefs were preferable to what was put in their place, but no one was listening to pagan nature-worshipers in Christian Europe. A third reason stems from the Judaeo-Christian sense of linear time. Other societies thought of time as
  • 55. cyclical, returning to earlier stages and starting over again. Linear time can be thought of as progressive or regressive, as moving on to better things or declining from some earlier, happier state. For Europeans in our period, the progressive view prevailed. In the last analysis, however, I would stress the role of the market: the fact that enterprise was free in Europe, that innovation worked and paid, that rulers and vested interests were narrowly constrained in what they could do to prevent or discourage innovation. Success bred imitation and emulation; also a sense of power that would in the long run raise men almost to the level of gods. The old legends remained—the expulsion from the Garden, Icarus who flew too high, Prometheus in chains—to warn against hubris. The very notion of hubris— cosmic insolence—is testimony to some men’s pretensions and the efforts of others to curb them. But the doers were not paying attention. The Second Chance: Learning from the Barbarians At the time the first Europeans arrived in the Indian Ocean and made their way to China, the Celestial Empire as it was called was, at least in its own eyes, the premier political entity in the world—first in size and population, first in age and experience, untouchable in its cultural achievement, apparently imperturbable in
  • 56. its sense of moral and spiritual superiority.5 The Chinese lived, as they thought, at the center of the universe; around them, lesser breeds basked in their glow, reached out to them for light, gained stature by doing obeisance and offering tribute. Their emperor was the “Son of Heaven,” the unique, godlike representative of celestial power. Those few who entered his presence showed their awe by kowtowing—kneeling and touching their head nine times to the ground; others kowtowed to anything emanating from him—a letter, a single handwritten ideo- graph. The paper he wrote on, the clothes he wore, everything he touched partook of his divine essence. Western diplomats allowed the Chinese to compel them to these gestures, which they “considered an essential part of a tributary system of foreign relations” (Spence, 1998, p. 42). By doing this, “the Westerners were 5 These Portuguese sailors of the sixteenth century were of course not the first Europeans to make their way to China. The best known of the earlier visitors is Marco Polo, who came in the thirteenth century from Venice, then the richest city in Europe, yet thought it a small town by comparison with what he saw in Cathay. 10 Journal of Economic Perspectives unwittingly shoring up the Qing court’s views of China’s
  • 57. superiority” (Spence citing Wills, 1984). Those who represented the emperor and administered for him were chosen on the basis of competitive examinations in Confucian letters and morals. These mandarin officials were in effect the embodiment of the higher Chinese culture, invested with its prestige, imbued with its wholeness and sublime superiority. Their self-esteem and haughtiness had ample room for expression and exercise on their inferiors and were matched only by their “stunned submissiveness” and self- abasement to superiors (Welsh, 1993, p. 16, who in this case quotes without reference). Nothing conveyed so well their rivalry in humility than the morning audience, when hundreds of courtiers gathered from midnight on and stood about in the open air, in rain and cold and fair, to wait for the emperor’s arrival and perform their obeisance. They were not wasting time; their time was the emperor’s. They could not afford to be late, and punctuality was not enough: unpunctual earliness was proof of zeal (Landes, 1983; see also Huang, 1981). Such cultural triumphalism combined with petty downward tyranny made China a singularly bad learner. What was there to learn? This rejection of the strange and foreign was the more anxious for the very force of the arrogance that
  • 58. justified it. For that is the paradox of the superiority complex: it is an expression of insecurity. It is intrinsically brittle; those who nourish it, need it, and depend on it are also those who fear nothing so much as contradiction. The French today are so persuaded of the superiority of their language that they dither and tremble at the prospect of a borrowed word, especially if it comes from English. The same holds for Ming China: they were so convinced of their ascendancy that they quaked before the challenge of Western technology, which was there for the learning. The irony is that those first Portuguese visitors and Catholic missionaries used the wonders of western technology to charm their way into China. The mechanical clock was the key that unlocked the gates. The mechanical clock was a European mega-invention of the late thirteenth century, crucial not only for its contribution to temporal discipline and productivity, but its susceptibility of improvement and its role at the frontier of instrumentation and mechanical technique. The water clock is a dunce by comparison. For the Chinese in the sixteenth century, the mechanical clock came as a wondrous machine capable not only of keeping time but of amusing and entertaining. Some clocks played music; others were automata with figurines that moved rhythmically at intervals. Clocks, then, were the sort of thing that the emperor would want to see, that had to be shown him if
  • 59. only to earn his favor, that a zealous courtier had to show him before someone else did. But that was not so easy. This magical device had to be accompanied. Where all Chinese instincts and practice dictated that foreigners should be kept at a distance, confined to some peripheral point like Macao and allowed to proceed to the center only by exception, the clock, in its sixteenth-century avatar, needed its attendant clock- maker and keepers. The Chinese loved clocks and watches. They were less happy, though, with their European attendants. The problem here was the Chinese sense of the whole- ness of culture, the link between things, people and the divine. The Catholic priests Why Europe and the West? Why Not China? 11 who first brought them these wonderful machines were salesmen of a special kind. They sought to convert the Chinese to the one true God, the trinitarian God of the Roman church, and the clocks were not only an entry ticket but an argument for the superiority of the Christian religion. Were not those who could make these things, who possessed all kinds of special astronomical and geographical knowledge to the bargain, were they not superior in the largest moral sense? Was not their faith
  • 60. truer, wiser? The Jesuits were prepared to make such an argument, stretching the while the rules and rites of the Church to fit the premises and win the sympathy of an understandably skeptical Chinese elite. (The Chinese ideographs for ancestor worship, for example, became the signifiers for the Christian mass.) But European laymen made the argument as well. Here is Gottfried Wilhelm von Leibniz (1646– 1716), mathematician (coinventor of the calculus) and philosopher (as quoted in Landes, 1983, p. 45, from a letter written circa 1675): What will these peoples say [the Persians, the Chinese], when they see this marvelous machine that you have made, which represents the true state of the heavens at any given time? I believe that they will recognize that the mind of man has something of the divine, and that this divinity communicates itself especially to Christians. The secret of the heavens, the greatness of the earth, and time measurement are the sort of thing I mean. This argument, whether explicit or implicit, did carry occasionally. The Cath- olic missionaries had some small success, although they had trouble persuading their open-minded “converts” to be good exclusivists (no other faith but the “true” faith) in the European tradition. But most Chinese saw these pretensions for what they were: an attack on Chinese claims to moral superiority, an assault on China’s
  • 61. self-esteem. The response, then, had to be a repudiation or depreciation of Western science and technology (Cipolla, 1967; Landes, 1983, chapter 2). Here is the K’ang Hsi emperor, the most open-minded and curious of men in his pursuit of Western ways, the most zealous in teaching them (as translated by Spence, 1974, p. 74): “[E]ven though some of the Western methods are different from our own, and may even be an improvement, there is little about them that is new. The principles of mathematics all derive from the Book of Changes, and the Western methods are Chinese in origin . . .” That was the heart-warming myth. So the Chinese, who were not prepared to give up clocks, who wanted clocks, who recognized their Western origin—these same Chinese trivialized clocks as toys (which for many they were) or as nonfunc- tional symbols of status, unaffordable by or inaccessible to most. Premodern imperial China did not think of time knowledge as a personal right. The hour was sounded by the authorities, and the right to own a timepiece was a rare privilege. As a result, although the imperial court set up workshops to make clocks and got their Jesuit clockmakers to train some native talent, these Chinese makers never arrived at the level of Western horologists—for want of the best teachers and lack
  • 62. of commercial competition and emulation. Nor did imperial China ever develop a 12 Journal of Economic Perspectives clockmaking trade comparable to that found in European countries. The same sin of pride (or indifference) shaped the Chinese response to European armament. Here was something that was anything but a toy. Cannons and muskets were instruments of death, hence of power, and the Chinese had every reason to interest themselves in these artifacts, the more so as the seventeenth century saw the progressive dissolution of the Ming dynasty and the conquest of China by a Tartar people from the north. These were decades of war, and the balance of power might well be tilted by access to these European inventions. Yet the Chinese never learned to make modern guns. Worse yet, they had known and used cannon as early as the thirteenth century but had forgotten much of what they had once known. Their city walls and gates had emplacements for cannon, but no cannon. Who needed them? The enemies of China did not have them. Yet China did have enemies, without and within, and no European nation would have been deterred from armament by enemy weakness; when it came to death, as in so many other things, the Europeans were
  • 63. maximizers. European technology was also monotonic-increasing: each gain was the basis for further gain. The Chinese record of advance followed by regression, step- forward, step-back, signaled an entirely different process. The Chinese, we are told, had a proverb: He who does not go forward will go backward (Peyrefitte, 1992, p. 157). The saying was apparently as much observation as prescription.6 So it was that in the seventeenth century, when the Portuguese in Macao offered three cannon to the emperor by way of gaining favor, they had to send three cannoneers along with them. Similarly, the Chinese hired on occasion Portuguese musketeers to do some fighting for them, and they got their Jesuit theologian-mechanicians to make them cannon. These cannon seem to have been among the best the Chinese had, so good compared to the run- of-the-foundry product that some were still in use in the nineteenth century, some 250 years later. If most Chinese guns did not last that long, it was because they were notoriously unreliable, more dangerous to the men who fired them than to the enemy. We even have one report of the use of clumps of dried mud as cannonballs. These at least had the merit of allowing the force of the explosion to exit by the mouth of the tube. In general, the authorities frowned on firearms, perhaps because they doubted the loyalty of their subjects (Cipolla, 1966, especially
  • 64. pp. 116–119).7 In view of the inefficacy of these pieces, one wonders what they had to fear. Presum- ably the improvement that comes with use. All of this may seem irrational to a means-ends oriented person, but it was not quite that; the ends were different. The European may have thought that the 6 Students of the history of Chinese technology and science, most notably Joseph Needham and his team, have made much of Chinese priority in discovery and invention, pushing the origins of important techniques and devices far back, well before their appearance in Europe. They see this quite properly as a sign of exceptional creativity and precocity, as discussed earlier in this paper, but they would do well then to ask why the subsequent retreat and loss. 7 Cipolla (1966) is not a sinologist and had to rely exclusively on European sources, including the testimony of Christian missionaries and travelers, but his “global vision” gives him crucial insights that are missing in the specialist literature. Guns, Sails, and Empires is a remarkable book. David S. Landes 13 purpose of war was to kill the enemy and win; the Chinese, strong in space and numbers, thought otherwise. Here is Mu Fu-sheng (1963, pp. 76–77, a pseudonym cited in Cipolla, 1966, p. 120) on the imperial viewpoint:
  • 65. . . . military defeat was the technical reason why Western knowledge should be acquired, but it was also the psychological reason why it should not be. Instinctively the Chinese preferred admitting military defeat, which could be reversed, to entering a psychological crisis; people could stand humiliation but not self-debasement . . . . The mandarins sensed the threat to Chinese civilization irrespective of the economic and political issues, and they tried to resist this threat without regard to the economic and political dangers. In the past the Chinese had never had to give up their cultural pride: the foreign rulers always adopted the Chinese civilization. Hence there was nothing in their history to guide them through their modern crisis. Along with Chinese indifference to technology went imperviousness to European science. The same conditions applied. The Jesuits and other Christian clerics brought in not only clocks but (sometimes obsolete) knowledge and ideas. Some of this was of interest to the court: in particular, astronomy and techniques of celestial observation were extremely valuable to a ruler who claimed a monopoly of the calendar and used his mastery of time to impose on the society as a whole. The Jesuits, moreover, trained gifted Chinese students who went on to do their own work: mathematicians who learned to use logarithms and trigonometry and astron-
  • 66. omers who prepared new star tables. Little of this got beyond Peking, however, and the pride some took in the new learning was soon countered by a nativist reaction that reached back to long- forgotten work of earlier periods. One leader of this return to the sources, Wen- Ting (1635–1721), examined the texts of mathematicians who had worked under the Song dynasty (10th–13th centuries) and proclaimed that the Jesuits had not brought much in the way of innovation. Later on, his manuscripts were published by his grandson under the title “Pearls Recovered from the Red River” (as discussed in Taton, 1963–1966, volume 2, p. 592). The title was more eloquent than intended: by this time much of Chinese scientific “inquiry” took the form of raking alluvial sediment. Meanwhile European science marched ahead, and successive churchmen brought to China better knowledge than their predecessors (though still well behind the frontier). Here, however, the churchmen were thwarted by the con- straints of their mission. The Christian missionaries had laid so much stress on the link between scientific knowledge and religious truth that any revision of the former implied a repudiation of the latter. When in 1710 a Jesuit astronomer sought to use new planetary tables based on the Copernican system, his superior
  • 67. would not permit it, for fear of “giving the impression of a censure on what our predecessors had so much trouble to establish and occasioning new accusations against [the Christian] religion” (Taton, 1963–1966, volume 2, p. 590). To recall these many instances of intellectual xenophobia is not to imply that 14 Journal of Economic Perspectives all Chinese were hostile to European ideas. We know that a few far-sighted officials and at least one emperor understood that the empire had much to gain by learning new ways.8 They were thwarted, however, not only by the studied complacency of an insecure superiority—also by a sense of completeness9—but by the intrigue of a palace milieu where innovations were judged by their consequences for the balance of power and influence. No proposals were made that did not incite resistance; no novelties offered that did not frighten vested interests. At all levels, moreover, fear of reprimand (or worse) outweighed the prospect of reward. A good idea brought credit to one’s superior; a mistake was invariably the fault of subordinates. One consequence was a prudent, almost instinctive, resistance to change. This is the heart of the matter: the response to difference and change.
  • 68. The Jesuit missionary Louis Le Comte (1655–1728) deplored this conservatism (as quoted in Cipolla, 1966, p. 120): “They are more fond of the most defective piece of antiquity than of the most perfect of the modern, differing much in that from us [Europe- ans], who are in love with nothing but what is new.” George Staunton, secretary to what is called the Macartney embassy from Great Britain to China from 1792 to 1794, disheartened by Chinese indifference to suggestions for improvement of their canals, lamented (Macartney, 1804, volume 6, p. 6), “In this country they think that everything is excellent and that proposals for improvement would be superfluous if not blameworthy.” A half-century later a Christian friar, Evariste Huc (1844–1846, volume 6, p. 81), discouraged perhaps by the sisyphean task of missionizing, despairingly observed: “Any man of genius is paralyzed immediately by the thought that his efforts will win him punishment rather than rewards.” Another consequence was a plague of lies and misinformation: officials wrote and told their superiors what they wanted to hear; or what the subordinate thought the superior would want to hear.10 The smothering of incentive and the cultivation of mendacity are characteristic weaknesses of large bureaucracies, whether public or private (business corporations). These are composed of nominal colleagues,
  • 69. who are supposedly pulling together but in fact are adversarial players. What is more, they compete within the organization, not in a free market of ideas, but in a closed world of guile and maneuver. Here the advantage lies with those in place. Reformers and subversives beware. The rejection of foreign technology was the more serious because China itself had long slipped into a regime of technological and scientific inertia, coasting along on the strength of previous gains and slowly losing speed as a result of the 8 The curse of foreignness remained though. In a letter of November 1640, the Jesuit von Bell wrote: “The word hsi [Western] is very unpopular, and the Emperor in his edicts never uses any word than hsin [new]; in fact the former word in used only by those who want to belittle us” (Taton, 1963–1966, volume 2, p. 589, n. 1). 9 For a discussion in this spirit, see Crone (1989, pp. 172–173): “China is a star example of a successful civilization. . . . China reached the pinnacle of economic development possible under pre-industrial conditions and stopped: no forces pushing it in a different direction are in evidence. . . .” 10 This is one of the major contributions of Peyrefitte’s (1992) book. Because he gained access to the Chinese archives, including papers read and annotated by the emperor, Peyrefitte is able to show the inner workings of bureaucratic equivocation and offer a valuable case study. Why Europe and the West? Why Not China? 15
  • 70. inevitable frictions of vested interest and diversion of talent and wealth into the comfort and gratification of gentility. It has been argued that such retirements from the fray should not deter ambitious newcomers; on the contrary, the prospect of happy exits should encourage entry, and departures should make room for others. But in most aristocratic societies, the availability of more esteemed careers seems to divert talent from commerce and industry by offering short cuts to high status. The withdrawal of successful merchants into land and office is seen as a logical promo- tion, a legitimate escape. In such circumstances, the presence of groups precluded by birth (thus merchants in Tokugawa Japan) or belief (Protestant dissenters in England) from access to office and honors—the existence, in other words, of a reserved pool of talent—may paradoxically be a strong contribution to otherwise inhibited economic development. Why Did China “Fail”? One of the great mysteries of Chinese history is why China did not produce from within the kind of scientific and industrial revolutions that gave Europe world dominion. A thousand years ago, the Chinese were well ahead of anyone else and certainly of Europe. Some would argue that this superiority held
  • 71. for centuries thereafter. Why, then, did China “fail”? Some China scholars would mitigate the pain by euphemism, as in Fairbank and Reischauer (1960, p. 291, cited in Oshima, 1987, p. 34): “Chinese society, though stable, was far from static and unchanging . . . the pace was slower . . . the degree of change less . . .”11 (True, but the issue remains.) Others would dismiss the question as unanswerable or illegitimate. Unanswerable because it is said to be impossible to explain a negative. (This is certainly not true in logic; the explanation of large-scale failure and success is inevitably complicated, but that is what history is all about.) Illegitimate because where is the failure? The very use of the word imposes non-Chinese standards and expectations on China. (But why not? Why should one not expect China to be interested in economic growth and develop- ment? To be curious about nature and want to understand it? To want to do more work with less labor? The earlier successes of China in these respects make these questions the more pertinent and acute.) What about the relations between science and technology? Did the one matter to the other? After all, science was not initially a major contributor to the European Industrial Revolution, which was built largely on empirical technological advances by practitioners. What difference, then, to Chinese practitioner
  • 72. technology if science had slowed to a crawl by the seventeenth century? The answer, I think, is that in both China and Europe, science and technology were (and are) two sides of the same coin, two manifestations of a common 11 Indeed, Fairbank and Reischauer (1960) suggest that the reason for Chinese “stability” was “the very perfection that Chinese culture and social organization had achieved by the thirteenth century.” The contrast with Europe, roiling with imperfection, could not be sharper. 16 Journal of Economic Perspectives approach to problems and experience. The response to new knowledge of either kind is of a piece, and the society that closes its eyes to novelty from one source has already been closing them to novelty from the other. In addition, China lacked the institutions that made for a cumulative process of finding and learning: the schools, the academies, the learned societies, the challenges and competitions. The sense of give-and-take, of standing on the shoul- ders of giants, of collective as well as individual achievement, of an inherited but ever imperfect treasure, of progress—all of these were weak or absent in China. And this is another paradox. On the one hand, the Chinese
  • 73. formally worshiped their intellectual ancestors; in 1734 an Imperial decree required court physicians to make ritual sacrifices to their departed predecessors (Taton, 1963–1966, volume 2, p. 590). On the other, the Chinese showed a deplorable tendency to let the findings of each new generation slip into oblivion, to be recovered perhaps at a later date by antiquarian and archaeological research.12 The history of Chinese advances, then, is one of points of light, separated in space and time, unlinked by replication and testing, obfuscated by metaphor and pseudo-profundity, limited in diffusion (with no technology for diffusion compa- rable to European printing)—in effect, a succession of ephemera. Much of the technical vocabulary was invented for the occasion and fell as swiftly into disuse; so that later scholars spent much of their effort trying to decipher these otherwise familiar ideograms. Much thought remained mired in metaphysical skepticism and speculation. Here Confucianism, with its easy disdain for scientific research, which it disparaged as “interventionist” and superficial, contributed its discouraging word. A poem written in the early nineteenth century by the son of the then–prime minister, himself a high state dignitary, warned (as quoted in Taton, 1963–1966, volume 2, p. 593): “With the microscope you see the surface of things. . . . But do not suppose you are seeing the things in themselves.”13
  • 74. The effect was discredit or indifference to science and technology, the greater for the want of mutual verification and support. This want of continuing intellec- tual exchange and reinforcement, this subjectivity, is what more than anything explains the uncertainty of scientific gains and the easy loss of impetus. Chinese savants had no way of knowing when they were right. It is subsequent research, mostly Western, that has discovered and awarded palms of achievement to the more inspired. Small wonder that China reacted so unfavorably to European imports. Euro- pean knowledge was not only strange and implicitly belittling. In its ebullience and excitement, its urgency and competitiveness, its brutal commitment to truth and efficacy (Jesuits excepted), it went against the Chinese mindset. 12 This ongoing slippage happened in spite of considerable effort to collect knowledge and present it in encyclopedias. One such project, really a kind of anthology, may well have been the biggest project of its kind ever attempted: 800,000 pages (Spence, 1990, p. 86). But a plethora of encyclopedias is a bad sign: like still photographs, they are an effort to fix knowledge at a point of time. They are useful as reference works, especially for historians, but they can impede free inquiry. 13 Of course, when the time came, one could find support in Confucianism for other positions. That is the nature of sacred writ: one can quote it to one’s purpose.
  • 75. David S. Landes 17 So the years passed, and the decades, and the centuries. China saw Europe leave it far behind. At first China was unbelieving and contemptuous. Later it became increasingly anxious and frustrated. From asking and begging, the West- erners became insistent and impatient. The British sent two embassies to China seeking improved trade relations: one headed by George Macartney in 1792 and a second headed by William Pitt Amherst in 1816. An underlying difficulty was that the Chinese were happy to sell to the British, but it was very difficult for the British to sell to the Chinese, except for silver and opium. After a series of diplomatic and trade confrontations, the First Opium War started in 1839. The British victory in that war resulted in the Treaty of Nanjing in 1842, which opened up Chinese ports to British ships, reduced Chinese tariffs on British goods, and ceded Hong Kong to the British. “There is Nothing We Lack” Now England is paying homage. My Ancestors’ merit and virtue must have reached their distant shores. Though their tribute is commonplace, my heart approves sincerely.
  • 76. Curios and the boasted ingenuity of their devices I prize not. Though what they bring is meager, yet, In my kindness to men from afar I make generous return, Wanting to preserve my good health and power. Poem by the Qienlong Emperor on the occasion of the Macartney embassy (1793) The Empire of China is an old, crazy, first rate man-of-war, which a fortunate succession of able and vigilant officers has contrived to keep afloat these one hundred and fifty years past, and to overawe their neighbours by her bulk and appearance, but whenever an insufficient man happens to have the command upon deck, adieu to the discipline and safety of the ship. She may perhaps not sink outright; she may drift some time as a wreck, and will then be dashed to pieces on the shore; but she can never be rebuilt on the old bottom. George, Lord Macartney to his journal (cited in Welsh, 1993, p. 33) The Chinese policy of superior indifference to Western things has been traditionally summed up in the dismissive letter of the Qienlong emperor (reigned 1736–1795) to George III, rejecting the British request of 1793 for trading rights and a permanent legation in Peking: “We have never set much store on strange and
  • 77. ingenious objects, nor do we need any more of your country’s manufactures.” So much for scientific instruments and technological devices. That is what I would call potent prose. It was by no means the only such contemptuous dismissal or trivial- 18 Journal of Economic Perspectives ization of foreign art and artifacts during these centuries of active contact (1550– 1900). Thus, the Qienlong Emperor’s successor, receiving and dismissing Macart- ney’s successor Lord Amherst in 1816, told him in effect to get lost: “My dynasty attaches no value to products from abroad; your nation’s cunningly wrought and strange wares do not appeal to me in the least” (as quoted in Sahlins, 1988, pp. 10–11). These explicit expressions of contempt, coming as they did from the emperor himself, leave little room for extenuation. The historian, even the apol- ogist, must deal with them—as the British had to. (They came back in 1839 with gunboats.) Yet the argument has now been put forward that these back-of- the-hand dismissals were not a rejection of Western knowledge, but rather messages for internal consumption. The Manchu dynasty then ruling China was foreign, its legitimacy open to question. It could not afford to nourish its
  • 78. enemies by admitting to a lack of autonomy, an inferiority to other outsiders. (This very fear of yielding— the definition of learning as weakness!—is testimony in my opinion to cultural defensiveness and introversion.) In fact, this thesis continues, the Chinese were very much interested in Western techniques and artifacts, especially in the military realm. What they did not want to import was European ideologies; and these two, technology and ideology, were closely linked. It was the Christian missionaries who had done that, using, as we have seen, European knowledge and devices to suggest the superiority of European religion (Waley-Cohen, 1993). But this argument is not sustained by the facts nor is it persuasive in logic. As to the facts: the Chinese long preceded the Europeans in the use of explosive powder, whether for display (fireworks) or use in weapons. Yet a study of their armament reveals a singular inability to enhance, by implication an indiffer- ence to, the destructive capacity of their bombards and cannon, to the point where they wreaked more fright than damage. Their very names bore witness to their inefficacy: thus we have the “nine-arrows, heart-penetrating, magically poisonous fire-thunderer,” a tube designed to blow a cluster of arrows in the direction of the enemy. Joseph Needham (1979) recognizes that these could not have gone very far, “since the gunpowder was not exerting its full propellant force.”
  • 79. But he conjectures that they might have some effect in close combat against lightly armored or unshielded personnel. Or the “eight-sided magical, awe- inspiring wind-and-fire cannon,” a vase-shaped bombard used to blow rubble and rubbish. Too bad those opposing these devices could not be told of their potent, magical, awe-inspiring names; they might have surrendered on the spot.14 Nor can one demonstrate a sustained and effective interest in European military technology by pointing to occasional instances of recourse to advice and 14 The Chinese use of hyperbole in describing weaponry seems to be a convention, and historians would be well advised to contain their credulity. We have an account of firearms and explosives in the later Ming period that speaks of cannon that “when they strike a city wall can reduce it instantly to rubble”; and of bombards whose sighting devices are so accurate that one “might pick off a general or remove a prince,” as quoted in Elvin (1973, p. 94). For critical comments on the value of this weaponry, see Sivin (1978, p. 468). Elvin in fact is reasonably skeptical, if only because he wants to know why the Chinese started so fast and then slowed down. Why Europe and the West? Why Not China? 19 technique from Jesuit missionaries. These good clerics were ready, in the cause of